Endocrine

¡Supera tus tareas y exámenes ahora con Quizwiz!

A client has returned to the nursing unit after a thyroidectomy. The nurse notes that the client is complaining of tingling sensations around the mouth, fingers, and toes. On the basis of these findings, the nurse should next assess the results of which serum laboratory study? A. Sodium B. Calcium C. Potassium D. Magnesium

B. Calcium After surgery on the thyroid gland, the client may experience a temporary calcium imbalance. This is due to transient malfunction of the parathyroid glands. The nurse also would assess for Chvostek's and Trousseau's signs. The correct treatment is administration of calcium gluconate or calcium lactate. Options 1, 3, and 4 are unrelated to the client's complaints.

The clinic nurse is providing instructions to a client with diabetes mellitus about the signs and symptoms of hypoglycemia. The nurse should tell the client that which would be noted in a hypoglycemic reaction? A. Thirst B. Hunger C. Polydipsia D. Increased urine output

B. Hunger Signs and symptoms of hypoglycemia include hunger, nervousness, anxiety, dizziness, blurred vision, sweaty palms, confusion, and tingling and numbness around the mouth. Polydipsia (thirst) and increased urine output are noted in the client with hyperglycemia.

A newly diagnosed client with diabetes mellitus is started on a two-dose insulin protocol combination of short- and intermediate-acting insulin injected twice daily. What portion of the total dose is given before breakfast and what portion before the evening meal? A. Half before breakfast and half before the evening meal B. Two thirds before breakfast and one third before the evening meal C. One third before breakfast and two thirds before the evening meal D. Three fourths before breakfast and one fourth before the evening meal

B. Two thirds before breakfast and one third before the evening meal Initially the two-dose insulin protocol is two thirds of the dose before breakfast and one third before the evening meal. Any future changes in these ratios are based on results of blood glucose monitoring. Therefore, options 1, 3, and 4 are incorrect procedure.

A client is admitted to the hospital with a diagnosis of pheochromocytoma. The nurse would check which item to detect the primary manifestation of this disorder? A. Weight B. Urine ketones C. Blood pressure D. Skin temperature

C. Blood pressure Hypertension is the major symptom associated with pheochromocytoma and is assessed by taking the client's blood pressure. Glycosuria, weight loss, and diaphoresis are other clinical manifestations of pheochromocytoma; however, hypertension is the major symptom.

A 33-year-old female client is admitted to the hospital with a tentative diagnosis of Graves' disease. Which symptom related to the menstrual cycle would the client be most likely to report during the initial assessment? A. Amenorrhea B. Menorrhagia C. Metrorrhagia D. Dysmenorrhea

A. Amenorrhea Amenorrhea or a decreased menstrual flow occurs in the client with Graves' disease. Menorrhagia, metrorrhagia and dysmenorrhea are also disorders related to the female reproductive system; however, they are not typical manifestations of Graves' disease.

A nurse is assessing the glycemic status of a client with diabetes mellitus. Which sign or symptom would indicate that the client is developing hyperglycemia? A. Polyuria B. Diaphoresis C. Hypertension D. Increased pulse rate

A. Polyuria Classic signs and symptoms of hyperglycemia include polydipsia, polyuria, and polyphagia. It is important to regularly assess the client for hyperglycemia to prevent the development of more serious complications, such as diabetic ketoacidosis. Options 2, 3, and 4 are not manifestations of hyperglycemia.

During routine nursing assessment after hypophysectomy, a client complains of thirst and frequent urination. Knowing the expected complications of this surgery, what should the nurse assess next? A. Serum glucose B. Blood pressure C. Respiratory rate D. Urine specific gravity

D. Urine specific gravity After hypophysectomy, temporary diabetes insipidus can result from antidiuretic hormone deficiency. This deficiency is related to surgical manipulation. The nurse should assess urine specific gravity, and notify the health care provider if the result is less than 1.005. Although options 1, 2, and 3 may be components of the assessment, the nurse would next assess urine specific gravity.

A client's serum blood glucose level is 48 mg/dL. The nurse would expect to note which as an additional finding when assessing this client? A. Slurred speech B. Increased thirst C. Increased appetite D. Increased urination

A. Slurred speech A client who has a blood glucose level of less than 70 mg/dL is considered to be hypoglycemic. A clinical manifestation of hypoglycemia is slurred speech.

A client with medullary carcinoma of the thyroid has an excess function of the C cells of the thyroid gland. The nurse plans care, knowing that this client is primarily at risk for abnormalities of which electrolytes? A. Sodium B. Calcium C. Potassium D. Magnesium

B. Calcium The C cells of the thyroid gland are helpful in maintaining normal plasma calcium levels. They do not affect the levels of sodium, potassium, or magnesium.

A nurse is caring for a client with pheochromocytoma. The client asks for a snack and something warm to drink. Which items would be the most appropriate choice for this client to meet nutritional needs? A. Crackers with cheese and tea B. Graham crackers and warm milk C. Toast with peanut butter and cocoa D. Vanilla wafers and coffee with cream and sugar

B. Graham crackers and warm milk The client with pheochromocytoma needs to be provided with a diet high in vitamins, minerals, and calories. Foods or beverages that contain caffeine, such as cocoa, coffee, tea, or colas are prohibited because they can precipitate a hypertensive crisis.

The nurse has provided dietary instructions to a client with a diagnosis of hypoparathyroidism. The nurse should instruct the client that it is acceptable to include which item in the diet? A. Fish B. Cereals C. Vegetables D. Meat and poultry

C. Vegetables The client with hypoparathyroidism is instructed to follow a calcium-rich diet and to restrict the amount of phosphorus in the diet. Vegetables are allowed in the diet. The client should limit meat, poultry, fish, eggs, cheese, and cereals.

The nurse is assessing a client who has a diagnosis of goiter. Which should the nurse expect to note during the assessment of the client? A. An enlarged thyroid gland B. The presence of heart damage C. Client complaints of chronic fatigue D. Client complaints of slow wound healing

A. An enlarged thyroid gland An enlarged thyroid gland develops in the client with goiter because of an excessive amount of thyroxine in the thyroid gland. Heart damage occurs with selenium deficiency. Additionally, heart damage would not likely be noted during the nursing assessment. Further diagnostic tests in addition to the assessment would be necessary to determine heart damage. Chronic fatigue occurs with iron deficiency. Slow wound healing occurs with zinc deficiency.

A client received 5 units of aspart insulin (NovoLog) subcutaneously just before eating lunch at 12:00 pm. The nurse should assess the client for a hypoglycemic reaction at which times? A. Between 1:00 and 3:00 pm B. 10 minutes after administration C. Between 4:00 pm and 12:00 am D. Between 8:00 pm and 10:00 pm

A. Between 1:00 and 3:00 pm Aspart insulin (NovoLog) is a rapid-acting insulin. Its onset of action is 15 minutes; it peaks in 1 to 3 hours, and its duration of action is 3 to 5 hours. Hypoglycemic reactions are most likely to occur during peak time.

The nurse is performing an assessment on a client with a diagnosis of myxedema (hypothyroidism). Which assessment finding should the nurse expect to note in this client? A. Dry skin B. Thin, silky hair C. Bulging eyeballs D. Fine muscle tremors

A. Dry skin Myxedema is a deficiency of thyroid hormone. The client will present with a puffy, edematous face, especially around the eyes (periorbital edema), along with coarse facial features, dry skin, and dry, coarse hair and eyebrows. Options 2, 3, and 4 are noted in the client with hyperthyroidism.

A nurse notes that a client with type 1 diabetes mellitus has lipodystrophy on both upper thighs. The nurse should ask the client if which measure is taken? A. Rotating sites for injection B. Administering the insulin at a 45-degree angle C. Cleaning the skin with alcohol before each injection D. Aspirating for blood before injection into the subcutaneous tissue

A. Rotating sites for injection Lipodystrophy (hypertrophy of subcutaneous tissue at the injection site) occurs in some clients with diabetes mellitus when injection sites are used for a prolonged period. Therefore, clients are instructed to adhere to a plan of rotating injection sites to avoid tissue changes. Angle of insulin administration, cleansing with alcohol, and aspiration do not produce this complication.

A nurse is assigned to care for a client with type 1 diabetes mellitus. During the shift, the nurse should monitor for which manifestation as a sign of hypoglycemia? A. Tremors B. Anorexia C. Hot, dry skin D. Muscle cramps

A. Tremors Decreased blood glucose levels trigger autonomic nervous system signs and symptoms, such as nervousness, irritability, and tremors. Hot, dry skin accompanies hyperglycemia. Anorexia and muscle cramps are unrelated to hypoglycemia.

A client has begun medication therapy with propylthiouracil (PTU). The nurse should assess the client for which condition as an adverse effect of this medication? A. Joint pain B. Renal toxicity C. Hyperglycemia D. Hypothyroidism

D. Hypothyroidism PTU is prescribed for the treatment of hyperthyroidism. Excessive dosing with this agent may convert a hyperthyroid state to a hypothyroid state. If this occurs, the dosage should be reduced. Temporary administration of thyroid hormone may be required to treat the hypothyroid state. PTU is not used for relief of joint pain. It does not cause renal toxicity or hyperglycemia.

A client with type 1 diabetes mellitus is admitted to the hospital with diabetic ketoacidosis and a serum glucose level of 789 mg/dL. The health care provider (HCP) prescribes 10 units of regular insulin by intravenous (IV) bolus, followed by a continuous insulin infusion at a rate of 5 units/hr. The pharmacy sends 500 mL of normal saline solution containing 50 units of regular insulin. After administering the IV bolus of 10 units of regular insulin, the nurse sets the infusion pump flow rate of the normal saline solution containing 50 units of regular insulin to infuse at how many milliliters per hour to deliver 5 units/hr? Fill in the blank. _____mL per hour

50 mL per hour This is a dosage calculation problem. Because the HCP does not prescribe a specific IV rate (mL/hr), the nurse needs to determine how many milliliters of the normal saline solution contain 5 units of insulin. The IV infusion pump is set at mL per hr. Use the ratio and proportion method for dosage calculation using the information contained in the question.

The health care provider prescribes a 24-hour urine collection for vanillylmandelic acid (VMA). The community health nurse visits the client at home and instructs the client in the procedure for the collection of the urine. Which statement, if made by the client, would indicate a need for further instruction? A. "I can take medication if I need to during the collection." B. "When I start the collection, I will urinate and discard that specimen." C. "I will pour the urine in the collection bottle each time I urinate and refrigerate the urine." D. "I will start the collection in 2 days. Starting now, I cannot eat or drink any tea, chocolate, vanilla, or fruit until the test is completed."

A. "I can take medication if I need to during the collection." Clients are reminded not to take medications for 2 to 3 days before a 24-hour urine collection for vanillylmandelic acid (VMA). Because a 24-hour urine collection is a timed quantitative determination, it is essential that the client start the test with an empty bladder. Therefore the client is instructed to void, discard the first urine, note the time, and start the test. The 24-hour urine specimen collection bottle must be kept on ice or refrigerated. For a VMA determination, the client is instructed to avoid tea, chocolate, vanilla, and all fruits for 2 days before urine collection begins.

The nurse is caring for a client who had a transsphenoidal hypophysectomy. Which statements should the nurse include in the discharge teaching instructions? Select all that apply. A. "Include adequate fiber and fluids in your diet." B. "Wear slip on shoes rather than those that need to be tied." C. "A post-nasal drip may be expected for several weeks after surgery." D. "Brushing your teeth will not be permitted for at least 2 weeks after surgery." E. "Contact your health care provider (HCP) immediately if you develop any headache, fever, or neck stiffness."

A. "Include adequate fiber and fluids in your diet." B. "Wear slip on shoes rather than those that need to be tied." D. "Brushing your teeth will not be permitted for at least 2 weeks after surgery." E. "Contact your health care provider (HCP) immediately if you develop any headache, fever, or neck stiffness." Clients who have undergone a transsphenoidal hypophysectomy will have an incision just above the upper lip so that pituitary gland can be accessed and removed through the sphenoid sinus. After the gland is removed, a muscle graft is taken, often from the thigh, to support the area and prevent leakage of cerebrospinal fluid (CSF). Clients should be taught to include adequate fluids and fiber in the diet to prevent straining during a bowel movement. Clients must also avoid bending from the waist to pick up objects or tie shoes because these activities will increase intracranial pressure. The client should also be taught to avoid brushing the teeth for 2 weeks to allow time for the incision to heal. Infection can occur after surgery so clients should be taught to immediately report headache, fever, and nuchal (neck) rigidity because these may be indicative of meningitis. Postnasal drip can be an indication of cerebrospinal fluid leak and should be immediately reported.

A client with diabetes mellitus is being tested to determine long-term diabetic control. Which result should the nurse expect to see if the client's long-term control is within acceptable limits? A. Glycosylated hemoglobin of 6% B. Presence of ketones in the urine C. Presence of albumin in the urine D. Fasting blood glucose level of 150 mg/dL

A. Glycosylated hemoglobin of 6% This measurement of glycosylated hemoglobin (Hb A1c) detects glucose binding on the red blood cell (RBC) membrane and is expressed as a percentage. It measures glucose for the life of the RBC, which is 120 days. A glycosylated hemoglobin of 6% is acceptable. The fasting blood glucose level should be at 110 mg/dL. The urine should be free of both ketones and urine.

A nurse is assessing the status of a client who returned to the surgical nursing unit after a parathyroidectomy procedure. The nurse would place highest priority on which assessment finding? A. Laryngeal stridor B. Difficulty voiding C. Mild incisional pain D. Absence of bowel sounds

A. Laryngeal stridor During the early postoperative period, the nurse carefully observes the client for signs of bleeding, which may cause swelling and compression of adjacent tissues. Laryngeal stridor results from compression of the trachea and is a harsh, high-pitched sound heard on inspiration and expiration. Laryngeal stridor is an acute emergency, necessitating immediate attention to avoid complete obstruction of the airway. The other options describe usual postoperative problems that are not life threatening.

A client with Graves' disease has exophthalmos and is experiencing photophobia. Which nursing action would best assist the client with these manifestations? A. Obtain dark glasses for the client. B. Lubricate the eyes with tap water every 2 to 4 hours. C. Administer methimazole (Tapazole) every 8 hours around the clock. D. Instruct the client to avoid straining or heavy lifting because this effort can increase eye pressure.

A. Obtain dark glasses for the client. Because photophobia (light intolerance) accompanies this disorder, wearing dark glasses is helpful in alleviating the problem. Tap water, which is hypotonic, could actually cause more swelling to the eye because it could pull fluid into the interstitial space. In addition, the client would be at risk for developing an eye infection because the solution is not sterile. Methimazole is a thyroid inhibitor, but medication therapy for Graves' disease does not help to alleviate the clinical manifestation of exophthalmos. There is no need to avoid straining or heavy lifting with exophthalmos.

During physical examination of a client, which finding is characteristic of hypothyroidism? A. Periorbital edema B. Flushed warm skin C. Hyperactive bowel sounds D. Heart rate of 120 beats/min

A. Periorbital edema Because cellular edema occurs in hypothyroidism, the client's appearance is changed. Nonpitting edema occurs, especially around the eyes and in the feet and hands. Knowing this should direct you to option 1. Options 2, 3, and 4 are clinical manifestations of hyperthyroidism, which occurs as a result of excess thyroid hormone secretion, resulting in a hypermetabolic state.

A client with suspected primary hyperparathyroidism is undergoing diagnostic testing. The nurse would assess for which as a manifestation of this disorder? A. Polyuria B. Diarrhea C. Polyphagia D. Weight gain

A. Polyuria Hypercalcemia classically occurs with hyperparathyroidism. Elevated serum calcium levels produce osmotic diuresis, making polyuria the correct option. The other manifestations listed are not associated with this disorder.

The nurse is caring for a client with a diagnosis of diabetic ketoacidosis (DKA). Which assessment findings are consistent with this diagnosis? Select all that apply. A. Polyuria B. Polydipsia C.Polyphagia D. Dry mouth E. Flushed, dry skin F. Moist mucous membranes

A. Polyuria B. Polydipsia C.Polyphagia D. Dry mouth E. Flushed, dry skin Clinical manifestations of DKA include polyuria (frequent urination), polydipsia (excessive thirst), polyphagia (excessive hunger), dry mouth, and flushed dry skin. The client with DKA experiences dehydration. Therefore option 6 would not be noted.

A client with an endocrine disorder complains of weight loss and diarrhea, and says that he can "feel his heart beating in his chest." The nurse interprets that which gland is most likely responsible for these symptoms? A. Thyroid B. Pituitary C. Parathyroid D. Adrenal cortex

A. Thyroid The thyroid gland is responsible for a number of metabolic functions in the body, including metabolism of nutrients (such as fats and carbohydrates). Increased metabolic function places a demand on the cardiovascular system for a higher cardiac output. Thus, a client with increased activity of the thyroid gland exhibits weight loss from higher metabolic rate and increased pulse rate.

A preoperative client is scheduled for adrenalectomy to remove a pheochromocytoma. The nurse would most closely monitor which item in the preoperative period? A. Vital signs B. Fluid balance C. Anxiety level D. Creatinine levels

A. Vital signs Hypertension is the hallmark symptom of pheochromocytoma. Severe hypertension can precipitate a stroke (brain attack) or sudden blindness. Although all of the items are appropriate nursing assessments for the client with pheochromocytoma, the priority is to monitor the vital signs, especially the blood pressure.

The nurse is taking a health history for a client with hyperparathyroidism. Which question would elicit information about this client's condition? A. "Do you have tremors in your hands?" B. "Are you experiencing pain in your joints?" C. "Do you notice swelling in your legs at night?" D. "Have you had problems with diarrhea lately?"

B. "Are you experiencing pain in your joints?" Hyperparathyroidism is associated with oversecretion of parathyroid hormone (PTH), which causes excessive osteoblast growth and activity within the bones. When bone reabsorption is increased, calcium is released from the bones into the blood, causing hypercalcemia. The bones suffer demineralization as a result of calcium loss, leading to bone and joint pain and, sometimes, pathological fractures. Options 1 and 4 relate to assessment of hypoparathyroidism. Option 3 is unrelated to hyperparathyroidism.

The nurse has provided instructions to the client with hyperparathyroidism regarding home care measures to manage the symptoms of the disease. Which statement by the client indicates a need for further instruction? A. "I should avoid bed rest." B. "I need to avoid doing any exercise at all." C. "I need to space activity throughout the day." D. "I should gauge my activity level by my energy level."

B. "I need to avoid doing any exercise at all." The client with hyperparathyroidism should pace activities throughout the day and plan for periods of uninterrupted rest. The client should plan for at least 30 minutes of walking each day to support calcium movement into the bones. The client should be instructed to avoid bed rest and use energy levels as a guide to activity. The client also should be instructed to avoid high-impact activity or contact sports.

The nurse is providing discharge instructions to a client who has Cushing's syndrome. Which client statement indicates that instructions related to dietary management are understood? A. "I will need to limit the amount of protein in my diet." B. "I should eat foods that have a lot of potassium in them." C. "I am fortunate that I can eat all the salty foods I enjoy." D. "I am fortunate that I do not need to follow any special diet."

B. "I should eat foods that have a lot of potassium in them." A diet low in carbohydrates and sodium but ample in protein and potassium is encouraged for a client with Cushing's syndrome. Such a diet promotes weight loss, reduction of edema and hypertension, control of hypokalemia, and rebuilding of wasted tissue.

The family of a bedridden client with type 2 diabetes mellitus and a chronic kidney disease calls a nurse to report symptoms of headache, polydipsia, and increased lethargy. Which most important question should the nurse ask the family to determine a possible problem? A. "What is the client's urine output?" B. "What is the client's capillary blood glucose level?" C. "Has there been any change in the dietary intake?" D. "Have you increased the amount of fluids provided?"

B. "What is the client's capillary blood glucose level?" Hyperglycemic hyperosmolar nonketotic syndrome (HHNS) is an acute complication of type 2 diabetes leading to hyperglycemia and dehydration. Headache, polydipsia, and increasing lethargy can be caused by the dehydration. The remaining options will not assist in determining a possible problem.

The client with pheochromocytoma is scheduled for surgery and says to the nurse, "I'm not sure that surgery is the best thing to do." Which statement is the appropriate response by the nurse? A. "I think you are making the right decision to have the surgery." B. "You have concerns about the surgical treatment for your condition?" C. "You are very ill. Your health care provider has made the correct decision." D. "There is no reason to worry. Your health care provider is a wonderful surgeon."

B. "You have concerns about the surgical treatment for your condition?" Paraphrasing is restating the client's messages in the nurse's own words. The correct option addresses the therapeutic communication technique of paraphrasing. Telling the client there is no reason to worry is offering a false reassurance, and this type of response will block communication. Telling the client that the health care provider has made the correct decision also represents a communication block in that it reflects lack of the client's right to an opinion. In the remaining option, the nurse is expressing approval, which can be harmful to the nurse-client relationship.

A client with diabetes mellitus has a blood glucose level of 50 mg/dL and reports feeling hungry and shaky. Which should the nurse provide the client? A. 3 oz of 2% milk B. 4 oz of apple juice C. 2 oz of orange juice D. A teaspoon of granulated sugar

B. 4 oz of apple juice When a client is exhibiting symptoms of mild hypoglycemia the nurse should provide the client with 15 g of a simple carbohydrate to quickly increase the blood glucose level. One half cup of apple juice is equivalent to 15 g of carbohydrates. The items in options 1, 3, and 4 do not provide a sufficient amount of carbohydrate.

The nurse is preparing to care for a client after parathyroidectomy. The nurse should plan for which action for this client? A. Maintain an endotracheal tube for 24 hours. B. Administer a continuous mist of room air or oxygen. C. Place in a flat position with the head and neck immobilized. D. Use only a rectal thermometer for temperature measurement.

B. Administer a continuous mist of room air or oxygen. Humidification of air or oxygen helps to liquefy mucous secretions and promotes easier breathing after parathyroidectomy. Pooling of thick mucus secretions in the trachea, bronchi, and lungs will cause respiratory obstruction. The client will not necessarily have an endotracheal tube in place. Tympanic temperatures can be taken. Semi-Fowler's position is the position of choice to assist in lung expansion and prevent edema. Rectal temperatures only are not required.

A client who visits the health care provider's office for a routine physical reports new onset of intolerance to cold. Knowing that this is a frequent complaint associated with hypothyroidism, the nurse should check for which manifestations? A. Weight loss and thinning skin B. Complaints of weakness and lethargy C. Diaphoresis and increased hair growth D. Increased heart rate and respiratory rate

B. Complaints of weakness and lethargy Weakness and lethargy are common complaints associated with hypothyroidism. Other common symptoms include weight gain, bradycardia, decreased respiratory rate, dry skin, and hair loss.

A client visits the health care provider's office for a routine physical examination and reports a new onset of intolerance to cold. Since hypothyroidism is suspected, which additional information would be noted during the client's assessment? A. Weight loss and tachycardia B. Complaints of weakness and lethargy C. Diaphoresis and increased hair growth D. Increased heart rate and respiratory rate

B. Complaints of weakness and lethargy Weakness and lethargy are the most common complaints associated with hypothyroidism. Other common symptoms include intolerance to cold, weight gain, bradycardia, decreased respiratory rate, dry skin, and hair loss.

A client with diabetes mellitus experiences breakdown of fats for conversion to glucose. The nurse determines that this response is occurring if the client has elevated levels of which substance? A. Glucose B. Ketones C. Glucagon D. Lactic dehydrogenase

B. Ketones Ketones are a byproduct of fat metabolism. When this process occurs to the extreme, it is termed ketoacidosis. Options 1, 3, and 4 are incorrect.

A client is admitted to an emergency department, and a diagnosis of myxedema coma is made. Which action would the nurse prepare to carry out initially? A. Warm the client. B. Maintain a patent airway. C. Administer thyroid hormone. D. Administer fluid replacement.

B. Maintain a patent airway. The initial nursing action would be to maintain a patent airway. Oxygen would be administered, followed by fluid replacement, keeping the client warm, monitoring vital signs, and administering thyroid hormones by the intravenous route.

A nurse understands that which hormone is directly responsible for maintaining the free or unbound portion of serum calcium within normal limits? A. Thyroid hormone B. Parathyroid hormone C. Follicle-stimulating hormone D. Adrenocorticotropic hormone

B. Parathyroid hormone Parathyroid hormone is responsible for maintaining serum calcium and phosphorous levels within normal range. Thyroid hormone is responsible for maintaining a normal metabolic rate in the body. Follicle-stimulating hormone and adrenocorticotropic hormone are produced by the anterior pituitary gland. They are responsible for growth and maturation of the ovarian follicle and stimulation of the adrenal glands, respectively.

A client with type 1 diabetes mellitus is admitted to the emergency department with suspected diabetic ketoacidosis (DKA). Which laboratory result would be expected with this diagnosis? A. Urine is negative for ketones. B. Serum potassium is 6.8 mEq/L. C. Serum osmolality is 260 mOsm/L. D. Arterial blood gas values are: pH 7.52, Pco2 44 mm Hg, HCO3 30 mEq/L.

B. Serum potassium is 6.8 mEq/L. Movement of hydrogen ions from the extracellular to the intracellular fluid promotes the movement of potassium from intracellular to extracellular fluid. Thus the serum potassium level will rise. The value in option 2 is greater than the normal range of 3.5 to 5.0 mEq/L. Presence of ketones in urine would be expected, and the serum osmolality would be elevated to reflect dehydration (the serum osmolality in option 3 is decreased). The client with DKA experiences metabolic acidosis (not metabolic alkalosis as noted in option 4).

A client is admitted to the hospital with a diagnosis of Addison's disease. The nurse would monitor for which problems associated with this disease? Select all that apply. A. Obesity B. Syncope C. Hirsutism D. Hypotension E. Muscle weakness

B. Syncope D. Hypotension E. Muscle weakness Common manifestations of Addison's disease include postural hypotension from fluid loss, syncope, muscle weakness, anorexia, nausea and vomiting, abdominal cramps, weight loss, depression, and irritability. Obesity and hirsutism do not occur with this disease.

A nurse is performing an admission assessment on a client with a diagnosis of pheochromocytoma. The nurse should assess for the major sign associated with pheochromocytoma by performing which action? A. Obtaining the client's weight B. Taking the client's blood pressure C. Testing the client's urine for glucose D. Palpating the skin for its temperature

B. Taking the client's blood pressure Pheochromocytoma is a catecholamine-producing tumor. Hypertension is the major sign associated with pheochromocytoma. Taking the client's blood pressure would assess the blood pressure status. Weight loss, glycosuria, and diaphoresis are also clinical manifestations of pheochromocytoma, yet hypertension is the major sign.

The nurse is preparing for a client's postoperative return to the unit after a parathyroidectomy procedure. The nurse should ensure that which piece of medical equipment is at the client's bedside? A. Cardiac monitor B. Tracheotomy set C. Intermittent gastric suction device D. Underwater seal chest drainage system

B. Tracheotomy set Respiratory distress caused by hemorrhage and swelling and compression of the trachea is a paramount concern for the nurse managing the care of a postoperative client who has had a parathyroidectomy. An emergency tracheotomy set is always routinely placed at the bedside of the client who has undergone this type of surgery, in anticipation of this complication. Options 1, 3, and 4 are not specifically needed with the surgical procedure.

Which findings should raise suspicion to the nurse that a head-injured client may be experiencing diabetes insipidus? Select all that apply. A. Ketones are present in the urine. B. Urine specific gravity is 1.001. C. Jugular venous distention is observed. D. Serum osmolality is 320 mOsm/kg of water. E. Blood glucose levels are greater than 200 mg/dL. F. Urine output has increased from 1000 mL in 24 hours to 4000 mL in 24 hours.

B. Urine specific gravity is 1.001. D. Serum osmolality is 320 mOsm/kg of water. F. Urine output has increased from 1000 mL in 24 hours to 4000 mL in 24 hours. Signs of diabetes insipidus include low urine specific gravity (<1.005), high serum osmolality (>300 mOsm/kg of water), and increased urine output from a deficiency of antidiuretic hormone (ADH). Options 1, 3, and 5 are not characteristic of diabetes insipidus.

A client tells the nurse that he enjoys outdoor gardening. The nurse understands that this client probably has active synthesis of which vitamin? A. Vitamin B B. Vitamin D C. Vitamin E D. Vitamin K

B. Vitamin D When exposed to ultraviolet light, the skin synthesizes vitamin D3, an intermediate form of vitamin D, which is then activated by the liver for metabolism of calcium and phosphorus. The other vitamins listed do not have this function.

The nurse is instructing a client with Cushing's syndrome on follow-up care. Which of these client statements would indicate a need for further instruction? A. "I should avoid contact sports." B. "I should check my ankles for swelling." C. "I need to avoid foods high in potassium." D. "I need to check my blood glucose regularly."

C. "I need to avoid foods high in potassium." Hypokalemia is a common characteristic of Cushing's syndrome, and the client is instructed to consume foods high in potassium. Clients with this condition experience activity intolerance, osteoporosis, and frequent bruising. Fluid volume excess results from water and sodium retention. Hyperglycemia is caused by an increased cortisol secretion.

The nurse is caring for a client who is scheduled to have a thyroidectomy and provides instructions to the client about the surgical procedure. Which client statement indicates an understanding of the nurse's instructions? A. "I expect to experience some tingling of my toes, fingers, and lips after surgery." B. "I will definitely have to continue taking antithyroid medications after this surgery." C. "I need to place my hands behind my neck when I have to cough or change positions." D. "I need to turn my head and neck front, back, and laterally every hour for the first 12 hours after surgery."

C. "I need to place my hands behind my neck when I have to cough or change positions." The client is taught that tension needs to be avoided on the suture line, otherwise hemorrhage may develop. One way of reducing incisional tension is to teach the client how to support the neck when coughing or being repositioned. Likewise, during the postoperative period the client should avoid any unnecessary movement of the neck. That is why sandbags and pillows frequently are used to support the head and neck. Any postoperative tingling in the fingers, toes, and lips probably is due to injury to the parathyroid gland during surgery, resulting in hypocalcemia. These signs and symptoms need to be reported immediately. Removal of the thyroid does not mean that the client will be taking antithyroid medications postoperatively. Thyroid replacement medications are necessary.

A home health nurse is visiting a client with type 1 diabetes mellitus. The client tells the nurse that he is not feeling well and has had a "respiratory infection" for the past week, which seems to be getting worse. After interviewing the client, what should be the initial nursing action? A. Notify the health care provider. B. Document the assessment data. C. Check the client's blood glucose. D. Obtain the client's sputum for culture and sensitivity.

C. Check the client's blood glucose. Uncontrolled hyperglycemia may lead to the production of ketones, thus leading to diabetic ketoacidosis (DKA), a life-threatening condition. The most common precipitating factor for development of diabetic ketoacidosis (DKA) is infection. Assessment data should be documented but are not a priority. The health care provider may need to be notified if the client's blood glucose is elevated and the client has other symptoms of DKA or a respiratory infection. After determining the client's blood glucose, the nurse should obtain a sputum sample if the client is expectorating yellow, green, or bloody secretions.

A nurse is caring for a client who had a thyroidectomy 1 day ago. Which client laboratory data should the nurse identify as a possible thyroid surgery complication? A. Increased serum sodium level B. Increased serum glucose level C. Decreased serum calcium level D. Decreased serum albumin level

C. Decreased serum calcium level Hypocalcemia may occur if the parathyroid glands are removed, damaged, or their blood supply is impaired during thyroid surgery, resulting in decreased parathyroid hormone (PTH) levels and lead to decreased serum calcium levels. Serum sodium, albumin, or glucose levels are not affected by thyroid surgery.

A nurse needs to maintain food and fluid intake to minimize the risk of dehydration in a client with diabetes mellitus who has gastroenteritis. Which is the appropriate nursing intervention? A. Offer water only until the client is able to tolerate solid foods. B. Withhold all fluids until vomiting has ceased for at least 4 hours. C. Encourage the client to take 8 to 12 oz of fluid every hour while awake. D. Maintain a clear liquid diet for at least 5 days before advancing to solids.

C. Encourage the client to take 8 to 12 oz of fluid every hour while awake. Small amounts of fluid may be tolerated, even when vomiting is present. The nurse should encourage liquids containing glucose and electrolytes every hour. The remaining options will not provide the adequate intake needed by the client with diabetes mellitus.

A nurse is reviewing the assessment findings and laboratory data for a client with the syndrome of inappropriate secretion of antidiuretic hormone (SIADH). The nurse understands that which symptoms are associated characteristics of this disorder? Select all that apply. A. Hypernatremia B. Signs of water deficit C. High urine osmolality D. Low serum osmolality E. Hypotonicity of body fluids F. Continued release of antidiuretic hormone

C. High urine osmolality D. Low serum osmolality E. Hypotonicity of body fluids F. Continued release of antidiuretic hormone SIADH is characterized by inappropriate continued release of antidiuretic hormone (ADH). This results in water intoxication, manifested as fluid volume expansion, hypotonicity of body fluids, and hyponatremia as a result of the high urine osmolality and low serum osmolality.

A nurse is caring for a client with a dysfunctional thyroid gland and is concerned that the client will exhibit signs of thyroid storm. Which is an early indicator of this complication? A. Constipation B. Bradycardia C. Hyperreflexia D. Low-grade temperature

C. Hyperreflexia Clinical manifestations of thyroid storm include a fever as high as 106° F, hyperreflexia, abdominal pain, diarrhea, dehydration rapidly progressing to coma, severe tachycardia, extreme vasodilation, hypotension, atrial fibrillation, and cardiovascular collapse.

A nurse is caring for a client with a thyrotoxicosis who is at risk for the development of thyroid storm. To detect this complication, the nurse should assess for which sign or symptom? A. Bradycardia B. Constipation C. Hypertension D. Low-grade temperature

C. Hypertension Thyroid storm is an acute, life-threatening condition that occurs in a client with uncontrollable hyperthyroidism. Clinical manifestations of thyroid storm include systolic hypertension, tachycardia, diarrhea, and a fever as high as 106° F. Other manifestations include abdominal pain, dehydration, extreme vasodilation, stupor rapidly progressing to coma, atrial fibrillation, and cardiovascular collapse. Options 1, 2, and 4 are not a part of the clinical picture in thyroid storm.

The nurse is caring for a client with a diagnosis of Cushing's syndrome. Which expected signs should the nurse monitor for? Select all that apply. A. Anorexia B. Dizziness C. Hypertension D. Weight loss E. Moon facies F. Truncal obesity

C. Hypertension E. Moon facies F. Truncal obesity A client with Cushing's syndrome may exhibit a number of different manifestations. These could include moon facies, truncal obesity, and a buffalo hump fat pad. Other signs include hypokalemia, peripheral edema, hypertension, increased appetite, and weight gain. Dizziness is not part of the clinical picture for this disorder.

A hospitalized client is experiencing an episode of hypoglycemia. The nurse plans care, knowing that which is the physiological mechanism that should take place to combat the decline in the blood glucose level? A. Decreased cortisol release B. Increased insulin secretion C. Increased glucagon secretion D. Decreased epinephrine release

C. Increased glucagon secretion Glucagon is secreted from the alpha cells in the pancreas in response to declining blood glucose levels. At the same time, hypoglycemia triggers increased cortisol release, decreased secretion of insulin, and increased epinephrine release.

A client's serum blood glucose level is 389 mg/dL. The nurse would expect to note which as an additional finding when assessing this client? A. Unsteady gait B. Slurred speech C. Increased thirst D. Cold, clammy skin

C. Increased thirst A clinical manifestation of hyperglycemia is increased thirst secondary to dehydration and frequent urination. Options 1, 2, and 4 would most likely be noted in hypoglycemia.

A nurse is assisting a client with diabetes mellitus who is recovering from diabetic ketoacidosis (DKA) to develop a plan to prevent a recurrence. Which is most important to include in the plan of care? A. Test urine for ketone levels. B. Eat six small meals per day. C. Monitor blood glucose levels frequently. D. Receive appropriate follow-up health care.

C. Monitor blood glucose levels frequently. Client education after DKA should emphasize the need for home glucose monitoring two to four times per day. Instructing the client to notify the health care provider when illness occurs is also important. The presence of urine ketones indicates that DKA has occurred already. The client should eat well-balanced meals with snacks as prescribed.

A client has been diagnosed with Cushing's syndrome. The nurse should assess this client for which expected manifestations of this disorder? A. Anorexia and weight loss B. Hypotension and dizziness C. Moon facies and truncal obesity D. Hyperkalemia and peripheral edema

C. Moon facies and truncal obesity The client with Cushing's syndrome may exhibit a number of different manifestations. These may include moon facies, truncal obesity, and a "buffalo hump" fat pad. Other signs include hypokalemia, peripheral edema, hypertension, increased appetite, and weight gain. Dizziness is not part of the clinical picture for this disorder.

A nurse is preparing to provide instructions to a client with Addison's disease regarding diet therapy. The nurse knows that which diet would most likely be prescribed for this client? A. High fat intake B. Low protein intake C. Normal sodium intake D. Low carbohydrate intake

C. Normal sodium intake A high-complex carbohydrate and high-protein diet will be prescribed for the client with Addison's disease. To prevent excess fluid and sodium loss, the client is instructed to maintain a normal salt intake daily (3 g) and to increase salt intake during hot weather, before strenuous exercise, and in response to fever, vomiting, or diarrhea. A high-fat diet is not prescribed.

A nurse is assessing a client who has had cranial surgery and is at risk for development of diabetes insipidus. The nurse would assess for which signs or symptoms that could indicate development of this complication? A. Diarrhea B. Infection C. Polydipsia D. Weight gain

C. Polydipsia Polydipsia and polyuria are classic symptoms of diabetes insipidus. The urine is pale, and the specific gravity is low. Diarrhea is not indicative of the complication. Infection is not associated with diabetes insipidus. Anorexia and weight loss also may occur.

A client with a history of diabetes mellitus has a fingerstick blood glucose level of 460 mg/dL. The home care nurse anticipates that which additional finding would be present with further testing if the client is experiencing diabetic ketoacidosis (DKA)? A. Hyponatremia B. Rise in serum pH C. Presence of ketone bodies D. Elevated serum bicarbonate level

C. Presence of ketone bodies DKA is marked by the presence of excessive ketone bodies. As a result of the acidosis, the pH and serum bicarbonate level would decrease. Hyponatremia is not related to DKA.

The nurse is caring for a postoperative client who has had an adrenalectomy. What should the nurse check for in the client's focused assessment? A. Peripheral edema B. Bilateral exophthalmos C. Signs and symptoms of hypovolemia D. Signs and symptoms of hypocalcemia

C. Signs and symptoms of hypovolemia Aldosterone, secreted by the adrenal cortex, plays a major role in fluid volume balance by retaining sodium and water. Thus, a deficiency can cause hypovolemia. A deficiency of adrenocortical hormones (such as after adrenalectomy) does not cause the clinical manifestations noted in options 1, 2, and 4.

A multidisciplinary health care team is developing a plan of care for a client with hyperparathyroidism. The nurse should include which priority intervention in the plan of care? A. Restrict fluids to 1000 mL per day. B. Describe the use of loperamide (Imodium). C. Walk down the hall for 15 minutes three times a day. D. Describe the administration of aluminum hydroxide gel.

C. Walk down the hall for 15 minutes three times a day. Mobility of the client with hyperparathyroidism should be encouraged as much as possible because of the calcium imbalance that occurs in this disorder and the predisposition to the formation of renal calculi. Fluids should not be restricted. Discussing the use of medications is not the priority with this client.

The nurse provides dietary instructions to a client with diabetes mellitus regarding the prescribed diet. Which statement, if made by the client, indicates a need for further teaching? A. "I'll eat a balanced meal plan." B. "I need to drink diet soft drinks." C. "I'll snack on fruit instead of cake." D. "I need to purchase special dietetic foods."

D. "I need to purchase special dietetic foods." It is important to emphasize to the client and family that they are not eating a diabetic diet but rather a balanced meal plan. Adherence to nutritional principles is an important component of diabetic management, and an individualized meal plan should be developed for the client. It is not necessary for the client to purchase special dietetic foods.

The nurse provides instructions to a client newly diagnosed with type 1 diabetes mellitus. The nurse recognizes accurate understanding of measures to prevent diabetic ketoacidosis when the client makes which statement? A. "I will stop taking my insulin if I'm too sick to eat." B. "I will decrease my insulin dose during times of illness." C. "I will adjust my insulin dose according to the level of glucose in my urine." D. "I will notify my health care provider (HCP) if my blood glucose level is higher than 250 mg/dL."

D. "I will notify my health care provider (HCP) if my blood glucose level is higher than 250 mg/dL." During illness, the client should monitor blood glucose levels and should notify the HCP if the level is higher than 250 mg/dL. Insulin should never be stopped. In fact, insulin may need to be increased during times of illness. Doses should not be adjusted without the HCP's advice and are usually adjusted on the basis of blood glucose levels, not urinary glucose readings.

A client with diabetes mellitus has been instructed in the dietary exchange system. The client asks the nurse if bacon is allowed in the diet. Which nursing response is most appropriate? A. "Bacon is not allowed." B. "Bacon is much too high in fat." C. "Bacon may be eaten if you eliminate one meat item from your diet." D. "One strip of bacon may be eaten if you eliminate 1 teaspoon of butter."

D. "One strip of bacon may be eaten if you eliminate 1 teaspoon of butter." Bacon is a component of the fat group in the exchange system. One teaspoon of butter is equal to 1 teaspoon of margarine, 1 teaspoon of any oil, 1 tablespoon of salad dressing, one strip of bacon, five large olives, or ten whole peanuts.

A client is undergoing an oral glucose tolerance test. The nurse interprets that the client's results are compatible with diabetes mellitus if the glucose level is at which value after 120 minutes (2 hours)? A. 80 mg/dL B. 110 mg/dL C. 130 mg/dL D. 160 mg/dL

D. 160 mg/dL The normal reference values for oral glucose tolerance tests are lower than 140 mg/dL at 120 minutes; lower than 200 mg/dL at 30, 60, and 90 minutes; and lower than 115 mg/dL in the fasting state. 160 mg/dL is higher than the normal reference range so therefore is the correct answer.

A nurse is preparing a teaching plan for a client with diabetes mellitus regarding proper foot care. Which instruction should be included in the plan? A. Soak the feet in hot water. B. Avoid using a mild soap on the feet. C. Always have a podiatrist cut the toenails. D. Apply a moisturizing lotion to dry feet but not between the toes.

D. Apply a moisturizing lotion to dry feet but not between the toes. The client is instructed to use a moisturizing lotion on the feet and avoid applying lotion between the toes. The client should be instructed not to soak the feet and should avoid hot water to prevent burns. The client should be instructed to wash the feet daily with a mild soap. The client may cut the toenails straight across and even with the toe itself and would consult a podiatrist if the toenails are thick or hard to cut or if vision is poor.

After hypophysectomy, a client complains of being thirsty and having to urinate frequently. What is the initial nursing action? A. Increase fluid intake. B. Document the complaints. C. Assess for urinary glucose. D. Assess urine specific gravity.

D. Assess urine specific gravity. After hypophysectomy, diabetes insipidus can occur temporarily because of antidiuretic hormone deficiency. This deficiency is related to surgical manipulation. The nurse should assess the specific gravity of the urine and notify the health care provider (HCP) if the result is lower than 1.006. Although to increase fluid intake and document the complaints may be components of the plan of care, they are not initial actions. Additionally, the HCP will prescribe increased fluids. Assessing for urinary glucose is unrelated to the client's condition.

The emergency department nurse is reviewing the laboratory test results for a client suspected of having diabetic ketoacidosis (DKA). Which laboratory result should the nurse expect to note in this disorder? A. Serum pH of 9.0 B. Absent ketones in the urine C. Serum bicarbonate of 22 mEq/L D. Blood glucose level of 500 mg/dL

D. Blood glucose level of 500 mg/dL In the client with DKA, the nurse should expect to note blood glucose levels between 350 and 1500 mg/dL, ketonuria, serum pH less than 7.35, and serum bicarbonate less than 15 mEq/dL.

A client is diagnosed with pheochromocytoma. The nurse understands that pheochromocytoma is a condition that has which characteristic? A. Causes profound hypotension B. Is manifested by severe hypoglycemia C. Is not curable and is treated symptomatically D. Causes the release of excessive amounts of catecholamines

D. Causes the release of excessive amounts of catecholamines Pheochromocytoma is a catecholamine-producing tumor and causes secretion of excessive amounts of epinephrine and norepinephrine. Hypertension is the principal manifestation, and the client has episodes of high blood pressure accompanied by pounding headaches. The excessive release of catecholamine also results in excessive conversion of glycogen into glucose in the liver. Consequently, hyperglycemia and glucosuria occur during attacks. Pheochromocytoma is curable. The primary treatment is surgical removal of one or both of the adrenal glands, depending on whether the tumor is unilateral or bilateral.

A hospitalized client is diagnosed with dysfunction of the adrenal medulla. The nurse monitors for changes in client status related to altered production and secretion of which substance? A. Cortisol B. Androgens C. Aldosterone D. Epinephrine

D. Epinephrine Epinephrine and norepinephrine are produced by the adrenal medulla. The other substances listed (cortisol, androgens, and aldosterone) are produced by the adrenal cortex.

A young man with type 1 diabetes mellitus tells the nurse that he might lose his job because he has been having frequent hypoglycemic reactions. His boss thinks that he is drunk during these episodes and that he has been drinking on the job. Which action by the nurse would best assist this client to meet his needs? A. Ask the client if he indeed has been drinking at work. B. Ask the client what he does to treat his hypoglycemia. C. Contact the local employment office to help him find another job. D. Examine factors with the client that may be causing frequent hypoglycemic episodes.

D. Examine factors with the client that may be causing frequent hypoglycemic episodes. Hypoglycemic reactions present as adrenergic symptoms of tremor, shakiness, and nervousness that are comparable or alike to the signs of alcohol intoxication. The best strategy to assist the client to meet his needs is to decrease the episodes of hypoglycemia by first identifying and then eliminating those factors that precipitate this event. Asking the client if he has been drinking at work and contacting the local employment office are inappropriate. Asking the client what he does to treat his hypoglycemia is not directly related to the subject of the question factors that may cause frequent hypoglycemic episodes.

A client with type 2 diabetes mellitus has a blood glucose level higher than 600 mg/dL and is complaining of polydipsia, polyuria, weight loss, and weakness. A nurse reviews the health care provider's documentation and should expect to note which diagnosis? A. Hypoglycemia B. Pheochromocytoma C. Diabetic ketoacidosis (DKA) D. Hyperglycemic hyperosmolar nonketotic syndrome (HHNS)

D. Hyperglycemic hyperosmolar nonketotic syndrome (HHNS) HHNS occurs in clients with type 2 diabetes mellitus. The onset of symptoms may be gradual. The symptoms may include polyuria, polydipsia, dehydration, mental status alterations, weight loss, and weakness. The remaining options are incorrect interpretations of the client's symptoms. Hypoglycemia is a complication of diabetes mellitus; manifestations include headache, nausea, sweating, tremors, lethargy, hunger, confusion, slurred speech, tingling around the mouth, and anxiety. Pheochromocytoma is a catecholamine-producing tumor and hypertension is the primary manifestation. Diabetic ketoacidosis (DKA) occurs in type 1 diabetes mellitus.

The nurse is reviewing the record of a client admitted to the hospital with a diagnosis of pheochromocytoma. The nurse reads the assessment findings and expects to note documentation of which major symptom associated with this condition? A. Glycosuria B. Diaphoresis C. Weight loss D. Hypertension

D. Hypertension Hypertension is the major symptom associated with pheochromocytoma. Glycosuria, weight loss, and diaphoresis also are clinical manifestations of pheochromocytoma; however, they are not major symptoms.

A client is admitted to the hospital with a diagnosis of Addison's disease. The nurse would assess for which problem as a manifestation of this disorder? A. Edema B. Obesity C. Hirsutism D. Hypotension

D. Hypotension Common manifestations of Addison's disease include postural hypotension from fluid loss, syncope, muscle weakness, anorexia, nausea and vomiting, abdominal cramps, weight loss, depression, and irritability. Options 1, 2, and 3 do not occur with this disease.

Which condition on assessment of the client with Addison's disease should the nurse expect to note? A. Edema B. Obesity C. Hirsutism D. Hypotension

D. Hypotension Common manifestations of Addison's disease include postural hypotension from fluid loss, syncope, muscle weakness, anorexia, nausea and vomiting, abdominal cramps, weight loss, depression, and irritability. The remaining options are not associated with this disorder.

A client is experiencing an episode of hypoglycemia. The nurse understands that the physiological mechanism that should take place to combat this decrease in the blood glucose level is which mechanism? A. Decreased cortisol release B. Increased insulin secretion C. Decreased epinephrine release D. Increased glucagon secretion

D. Increased glucagon secretion Glucagon is secreted from the alpha cells in the pancreas in response to declining blood glucose levels. At the same time, hypoglycemia triggers increased cortisol release, increased epinephrine release, and decreased secretion of insulin. Options 1, 2, and 3 are not physiological mechanisms that take place to combat the decrease in the blood glucose level.

A client who has had intracranial surgery is experiencing diabetes insipidus. The nurse understands that the diabetes insipidus resulted from which problem? A. Water intoxication B. Excess production of dopamine C. Excess production of angiotensin II D. Insufficient production of antidiuretic hormone (ADH)

D. Insufficient production of antidiuretic hormone (ADH) Diabetes insipidus results from insufficient ADH production, which in this case was because of head injury. This causes the kidneys to excrete large volumes of urine. Water intoxication represents the opposite problem of that experienced with diabetes insipidus. Options 2 and 3 are not associated with diabetes insipidus.

A nurse is performing an assessment on a client after a thyroidectomy and notes that the client has developed hoarseness and a weak voice. Which nursing action is appropriate? A. Check for signs of bleeding. B. Administer calcium gluconate. C. Notify the health care provider (HCP) immediately. D. Reassure the client that this is usually a temporary condition.

D. Reassure the client that this is usually a temporary condition. After thyroidectomy, weakness and hoarseness of the voice can occur as a result of trauma from the surgery. If this develops, the client should be reassured that the problem will subside in a few days. Unnecessary talking should be discouraged. These signs do not indicate bleeding or the need to administer calcium gluconate. The nurse does not need to notify the HCP immediately.

A health care provider has prescribed propylthiouracil (PTU) for a client with hyperthyroidism, and the nurse develops a plan of care for the client. The nurse should assess for which condition as a priority? A. Relief of pain B. Signs of renal toxicity C. Signs of hyperglycemia D. Signs of hypothyroidism

D. Signs of hypothyroidism Excessive dosing with PTU may convert the client from a hyperthyroid state to a hypothyroid state. If this occurs, the dosage should be reduced. Temporary administration of thyroid hormone may be required. PTU is not used for pain and does not cause renal toxicity or hyperglycemia.

A client newly diagnosed with diabetes mellitus has been stabilized with daily insulin injections. A nurse prepares a discharge teaching plan regarding the insulin and plans to reinforce which concept? A. Always keep insulin vials refrigerated. B. Ketones in the urine signify a need for less insulin. C. Increase the amount of insulin before excessive exercise. D. Systematically rotate insulin injections within one anatomical site.

D. Systematically rotate insulin injections within one anatomical site. Injection sites should be rotated systematically within one anatomical site. To minimize the discomfort associated with insulin injections, insulin should be administered at room temperature. If ketones are found in the urine, it may indicate the need for additional insulin. Insulin doses should not be adjusted or increased before excessive exercise.

The nurse is assessing the learning readiness of a client newly diagnosed with diabetes mellitus. Which behavior indicates to the nurse that the client is not ready to learn? A. The client asks if the spouse may attend the teaching session. B. The client asks appropriate questions about what will be taught. C. The client asks for written materials about diabetes mellitus before class. D. The client complains of fatigue whenever the nurse plans a teaching session.

D. The client complains of fatigue whenever the nurse plans a teaching session. Physical symptoms can interfere with an individual's ability to learn and can indicate to the teacher that the student lacks motivation to learn if the symptoms repeatedly recur when teaching is initiated. The remaining options identify active client participation in learning.

A client with diabetes mellitus who takes insulin is seen in the health care clinic. The client tells the clinic nurse that after the insulin injection, the insulin seems to leak through the skin. The nurse would appropriately determine the problem by asking the client which question? A. "Are you rotating the injection site?" B. "Are you aspirating before you inject the insulin?" C. "Are you using a 1-inch needle to give the injection?" D. "Are you placing an air bubble in the syringe before injection?"

A. "Are you rotating the injection site?" The client should be instructed that insulin injection sites should be rotated within one anatomical area before moving on to another area. This rotation process promotes uniform absorption of insulin and reduces the chances of irritation. Options 2, 3, and 4 are not associated with the condition (skin leakage of insulin) presented in the question.

A registered nurse (RN) is caring for a client with a diagnosis of Cushing's syndrome. A nursing student is working with the RN for the day. Which statement by the student indicates understanding of Cushing's syndrome? A. "Cushing's syndrome is caused by excessive amounts of cortisol." B. "Cushing's syndrome is caused by decreased amounts of aldosterone." C. "Cushing's syndrome is caused by excessive amounts of antidiuretic hormone." D. "Cushing's syndrome is caused by decreased amounts of parathyroid hormone."

A. "Cushing's syndrome is caused by excessive amounts of cortisol." Cushing's syndrome is a condition caused by excessive amounts of cortisol. Options 2, 3, and 4 are inaccurate descriptions of this disorder.

The home health nurse visits a client with a diagnosis of type 1 diabetes mellitus. The client relates a history of vomiting and diarrhea and tells the nurse that no food has been consumed for the last 24 hours. Which additional statement by the client indicates a need for further teaching? A. "I need to stop my insulin." B. "I need to increase my fluid intake." C. "I need to monitor my blood glucose every 3 to 4 hours." D. "I need to call the health care provider (HCP) because of these symptoms."

A. "I need to stop my insulin." When a client with diabetes mellitus is unable to eat normally because of illness, the client still should take the prescribed insulin or oral medication. The client should consume additional fluids and should notify the HCP. The client should monitor the blood glucose level every 3 to 4 hours. The client should also monitor the urine for ketones.

A client has been hospitalized for an endocrine system dysfunction of the pancreas. The nurse providing care for the client anticipates that he or she will exhibit impaired secretion of which substances? A. Insulin B. Lipase C. Trypsin D. Amylase

A. Insulin The pancreas produces both endocrine and exocrine secretions as part of its normal function. The organ secretes insulin as a key endocrine hormone to regulate the blood glucose level. Other pancreatic endocrine hormones are glucagon and somatostatin. The exocrine pancreas produces digestive enzymes such as amylase, lipase, and trypsin.

The nurse is developing a plan of care for a client with Cushing's syndrome. The nurse documents a client problem of excess fluid volume. Which nursing actions should be included in the care plan for this client? Select all that apply. A. Monitor daily weight. B. Monitor intake and output. C. Assess extremities for edema. D. Maintain a high-sodium diet. E. Maintain a low-potassium diet.

A. Monitor daily weight. B. Monitor intake and output. C. Assess extremities for edema. The client with Cushing's syndrome and a problem of excess fluid volume should be maintained on a high-potassium, low-sodium diet. Decreased sodium intake decreases renal retention of sodium and water. Options 1, 2, and 3 are appropriate interventions for the client with excess fluid volume.

The nurse is providing home care instructions to the client with a diagnosis of Cushing's syndrome and prepares a list of instructions for the client. Which instructions should be included on the list? Select all that apply. A. The signs and symptoms of hypoadrenalism B. The signs and symptoms of hyperadrenalism C. Instructions to take the medications exactly as prescribed D. The importance of maintaining regular outpatient follow-up care E. A reminder to read the labels on over-the-counter medications before purchase

A. The signs and symptoms of hypoadrenalism B. The signs and symptoms of hyperadrenalism C. Instructions to take the medications exactly as prescribed D. The importance of maintaining regular outpatient follow-up care The client with Cushing's syndrome should be instructed to take the medications exactly as prescribed. The nurse should emphasize the importance of continuing medications, consulting with the health care provider (HCP) before purchasing any over-the-counter medications, and maintaining regular outpatient follow-up care. The nurse also should instruct the client in the signs and symptoms of both hypoadrenalism and hyperadrenalism.

A client with an endocrine disorder has experienced recent weight loss and exhibits tachycardia. The nurse plans care, knowing that which gland is most likely to be responsible for these findings? A. Thyroid B. Pituitary C. Parathyroid D. Adrenal cortex

A. Thyroid The thyroid gland is responsible for a number of metabolic functions in the body. Among these are metabolism of nutrients such as fats and carbohydrates. Increased metabolic function places a demand on the cardiovascular system for a higher cardiac output. A client with increased activity of the thyroid gland will experience weight loss from the higher metabolic rate and will have an increased pulse rate. The anterior pituitary gland produces GH, LH, and FSH. ADH and oxytocin are secreted by the posterior pituitary gland. Both ADH and oxytocin are synthesized by the hypothalamus and stored in the posterior pituitary gland. These hormones are released as needed into the bloodstream. Parathyroid hormone is responsible for maintaining serum calcium and phosphorus levels within normal range. The adrenal cortex is responsible for the production of glucocorticoids and mineralocorticoids.

A client has been hospitalized for impaired function of the posterior pituitary gland. The nurse providing care for the client anticipates that he or she may exhibit altered secretion of which hormones? A. Growth hormone (GH) B. Luteinizing hormone (LH) C. Antidiuretic hormone (ADH) D. Follicle-stimulating hormone (FSH)

C. Antidiuretic hormone (ADH) ADH is secreted by the posterior pituitary gland. The other hormone stored in the posterior pituitary gland is oxytocin. Both ADH and oxytocin are synthesized by the hypothalamus and stored in the posterior pituitary gland. These hormones are released as needed into the bloodstream. The anterior pituitary gland produces GH, LH, and FSH.

The nurse has provided home care measures to the client with diabetes mellitus regarding exercise and insulin administration. Which statement by the client indicates a need for further instruction? A. "I should always wear a Medic-Alert bracelet." B. "I should perform my exercise at peak insulin time." C. "I should always carry a quick-acting carbohydrate when I exercise." D. "I should avoid exercising at times when a hypoglycemic reaction is likely to occur."

B. "I should perform my exercise at peak insulin time." The client should be instructed to avoid exercise at peak insulin time because this is when a hypoglycemic reaction is likely to occur. If exercises are performed at this time, the client should be instructed to eat an hour before the exercise and drink a carbohydrate liquid. Options 1, 3, and 4 are correct statements regarding exercise, insulin, and diabetic control.

A client with Cushing's syndrome verbalizes concern to the nurse regarding the appearance of the buffalo hump that has developed. Which statement should the nurse make to the client? A. "Don't be concerned; this problem can be covered with clothing." B. "Usually these physical changes slowly improve following treatment." C. "This is permanent, but looks are deceiving and are not that important." D. "Try not to worry about it; there are other things to be concerned about."

B. "Usually these physical changes slowly improve following treatment." The client with Cushing's syndrome should be reassured that most physical changes resolve with treatment. All other options are not therapeutic responses.

A client's serum calcium level is high. The nurse plans care knowing that which hormones are directly responsible for maintaining the free or unbound portion of the serum calcium within normal limits? A. Thyroid hormone B. Parathyroid hormone C. Follicle-stimulating hormone D. Adrenocorticotropic hormone

B. Parathyroid hormone Parathyroid hormone is responsible for maintaining serum calcium and phosphorus levels within normal range. Thyroid hormone is responsible for maintaining a normal metabolic rate in the body. Follicle-stimulating hormone and adrenocorticotropic hormone are produced by the anterior pituitary gland. They are responsible for growth and maturation of the ovarian follicle and stimulation of the adrenal glands, respectively.

A client with suspected Cushing's syndrome is scheduled for adrenal venography. A nurse has provided instructions to the client regarding the test. Which statement by the client indicates a need for further instruction? A. "I need to sign an informed consent." B. "The insertion site will be locally anesthetized." C. "I will be placed in a high-sitting position for the test." D. "I may feel a burning sensation after the dye is injected."

C. "I will be placed in a high-sitting position for the test." The test aids in determining whether signs and symptoms are caused by abnormalities in the adrenal gland. The nurse assesses the client for allergies to iodine before the test. The client is informed that that the supine position is necessary to access the femoral vein. An informed consent form is required, the insertion site will be locally anesthetized, and the client will experience a transient burning sensation after the dye is injected.

A nurse is providing home care instructions to a client with a diagnosis of Addison's disease. Which statement by the client indicates a need for further instruction? A. "I need to wear a Medic-Alert bracelet." B. "I need to purchase a travel kit that contains cortisone." C. "I will need to take daily medications until my symptoms decrease." D. "I need an increased dose of glucocorticoid medication during stressful minor illnesses."

C. "I will need to take daily medications until my symptoms decrease." Client teaching includes the need for lifelong daily medications. The client also is instructed to carry or wear a medical identification card or bracelet. A travel kit will need to be purchased. It should contain oral cortisone along with intramuscular preparations for self-injection and intravenous vials for emergency injection by a health care provider. Increased glucocorticoid dosage during stressful minor illnesses will be necessary.

A client with hypovolemia experiences activation of the renin-angiotensin system to maintain blood pressure. The nurse plans care, understanding that, as part of this response, the endocrine system will increase production and secretion of which mineralocorticoid? A. Cortisol B. Glucagon C. Aldosterone D. Adrenocorticotropic hormone

C. Aldosterone Aldosterone is the primary mineralocorticoid that is produced and secreted in response to lowered blood volume. Cortisol is a glucocorticoid. Glucagon is produced by the pancreas and functions to oppose the action of insulin in regulating blood glucose levels. Adrenocorticotropic hormone is produced by the pituitary gland and stimulates the adrenal cortex to produce glucocorticoids and mineralocorticoids.

The nurse is caring for a client after thyroidectomy. The nurse notes that calcium gluconate is prescribed for the client. The nurse determines that this medication has been prescribed for which purpose? A. To treat thyroid storm B. To prevent cardiac irritability C. To treat hypocalcemic tetany D. To stimulate release of parathyroid hormone

C. To treat hypocalcemic tetany Hypocalcemia, resulting in tetany, can develop after thyroidectomy if the parathyroid glands are accidentally removed during surgery. Manifestations develop 1 to 7 days after surgery. If the client develops numbness and tingling around the mouth, fingertips, or toes; muscle spasms; or twitching, the health care provider is notified immediately. Calcium gluconate should be readily available in the nursing unit.

The nurse is caring for a client admitted to the emergency department with diabetic ketoacidosis (DKA). In the acute phase, the nurse plans for which priority intervention? A. Correct the acidosis. B. Administer 5% dextrose intravenously. C. Apply a monitor for an electrocardiogram. D. Administer short-duration insulin intravenously.

D. Administer short-duration insulin intravenously. Lack (absolute or relative) of insulin is the primary cause of DKA. Treatment consists of insulin administration (short or rapid-acting), intravenous fluid administration (normal saline initially), and potassium replacement, followed by correcting acidosis. Applying an electrocardiogram monitor is not the priority action.

The nurse is caring for a client scheduled for a transsphenoidal hypophysectomy. The preoperative teaching instructions should include which most important statement? A. "Your hair will need to be shaved." B. "You will receive spinal anesthesia." C. "You will need to ambulate after surgery." D. "Brushing your teeth will not be permitted for at least 2 weeks after surgery."

D. "Brushing your teeth will not be permitted for at least 2 weeks after surgery." Based on the location of the surgical procedure, spinal anesthesia would not be used. In addition, the hair would not be shaved. Although ambulating is important, specific to this procedure is avoiding brushing the teeth to prevent disruption of the surgical site.

A test to measure long-term control of diabetes mellitus has been prescribed for a client. In instructing the client about the test, the nurse explains that long-term control can be measured because chronic high blood glucose levels lead to irreversible glucose binding onto what? A. Platelets B. Muscle tissue C. Adipose tissue D. Red blood cells

D. Red blood cells With chronic high circulating blood glucose levels, some glucose binds irreversibly onto red blood cells (RBCs) and remains there for the life of the cell. The average life span of an RBC is 120 days. The measurement of glycosylated hemoglobin (HbA1c), which detects glucose binding on the RBC membrane, is expressed as a percentage. Glucose does not bind onto platelets in diabetes mellitus. One of the problems in diabetes is that muscle and adipose cells may be unable to transport glucose across cell membranes.

The nurse is caring for a client with a diagnosis of Addison's disease. The nurse is monitoring the client for signs of Addisonian crisis. The nurse should assess the client for which manifestation that would be associated with this crisis? A. Agitation B. Diaphoresis C. Restlessness D. Severe abdominal pain

D. Severe abdominal pain Addisonian crisis is a serious life-threatening response to acute adrenal insufficiency that most commonly is precipitated by a major stressor. The client in Addisonian crisis may demonstrate any of the signs and symptoms of Addison's disease, but the primary problems are sudden profound weakness; severe abdominal, back, and leg pain; hyperpyrexia followed by hypothermia; peripheral vascular collapse; coma; and renal failure. Options 1, 2, and 3 do not identify clinical manifestations associated with Addisonian crisis.

The nurse is providing dietary instructions to help with diabetes control for a client newly diagnosed with diabetes mellitus who will be taking insulin. The nurse should provide the client with which best instruction? A. Eat meals at approximately the same time each day. B. Adjust meal times depending on blood glucose levels. C. Vary meal times if insulin is not administered at the same time every day. D. Avoid being concerned about the time of meals so long as snacks are taken on time.

A. Eat meals at approximately the same time each day. Meal times must be approximately the same each day to maintain a stable blood glucose level. The client should not be instructed that meal times can be varied depending on blood glucose levels, insulin administration, or consumption of snacks.

A client has been diagnosed with hyperthyroidism. Which signs and symptoms may indicate thyroid storm, a complication of this disorder? Select all that apply. A. Fever B. Nausea C. Lethargy D. Tremors E. Confusion F. Bradycardia

A. Fever B. Nausea D. Tremors E. Confusion Thyroid storm is an acute and life-threatening condition that occurs in a client with uncontrollable hyperthyroidism. Symptoms of thyroid storm include elevated temperature (fever), nausea, and tremors. In addition, as the condition progresses, the client becomes confused. The client is restless and anxious and experiences tachycardia.

The nurse is monitoring a client with Graves' disease for signs of thyrotoxic crisis (thyroid storm). Which signs or symptoms, if noted in the client, will alert the nurse to the presence of this crisis? A. Fever and tachycardia B. Pallor and tachycardia C. Agitation and bradycardia D. Restlessness and bradycardia

A. Fever and tachycardia Thyrotoxic crisis (thyroid storm) is an acute, potentially life-threatening state of extreme thyroid activity that represents a breakdown in the body's tolerance to a chronic excess of thyroid hormones. The clinical manifestations include fever with temperatures greater than 100° F, severe tachycardia, flushing and sweating, and marked agitation and restlessness. Delirium and coma can occur.

A client arrives in the hospital emergency department in an unconscious state. As reported by the spouse, the client has diabetes mellitus and began to show symptoms of hypoglycemia. A blood glucose level is obtained for the client, and the result is 40 mg/dL. Which medication should the nurse anticipate to be prescribed for the client? A. Glucagon B. Humulin N insulin C. Humulin R insulin D. Glyburide (DiaBeta)

A. Glucagon A blood glucose lower that 50 mg/dL is considered to be critically low. Glucagon is used to treat hypoglycemia because it increases blood glucose levels. Humulin N insulin and Humulin R insulin would lower the client's blood glucose and would not be an appropriate treatment for hypoglycemia. Glyburide is an oral hypoglycemic agent used to treat type 2 diabetes mellitus and would not be given to a client with hypoglycemia. Additionally, an oral medication would not be administered to an unconscious client.

A client with diabetes mellitus who refuses to take insulin as prescribed exhibits markedly increased blood glucose levels after a meal. The nurse caring for the client anticipates that which initial body response to elevated glucose levels will worsen the situation for the client? A. Glycogenolysis B. Gluconeogenesis C. Binding of glucose onto cell membranes D. Transport of glucose across cell membranes

A. Glycogenolysis As blood glucose levels rise when glucose is not being carried into the cells, the body interprets this to mean that more glucose is needed. The initial response by the body is to use up the stores of glycogen in the liver. The conversion of glycogen to glucose for use by the body is called glycogenolysis. If this mechanism fails, the body breaks down fats and proteins and converts them into glucose; this process is called gluconeogenesis. Glucose binds onto cell membranes and is transported across them into the cells when there is sufficient circulating insulin.

The nurse is caring for a client who has had an adrenalectomy and is monitoring the client for signs of adrenal insufficiency. Which signs and symptoms indicate adrenal insufficiency in this client? A. Hypotension and fever B. Mental status changes and hypertension C. Subnormal temperature and hypotension D. Complaints of weakness and hypertension

A. Hypotension and fever The nurse should be alert to signs and symptoms of adrenal insufficiency after adrenalectomy. These signs and symptoms include weakness, hypotension, fever, and mental status changes. Options 2, 3, and 4 are incorrect options.

The nurse is monitoring a client who was diagnosed with type 1 diabetes mellitus and is being treated with NPH and regular insulin. Which client complaints would alert the nurse to the presence of a possible hypoglycemic reaction? Select all that apply. A. Tremors B. Anorexia C. Irritability D. Nervousness E. Hot, dry skin F. Muscle cramps

A. Tremors C. Irritability D. Nervousness Decreased blood glucose levels produce autonomic nervous system symptoms, which are manifested classically as nervousness, irritability, and tremors. Option E is more likely to occur with hyperglycemia. Options B and F are unrelated to the signs of hypoglycemia. In hypoglycemia, usually the client feels hunger.

A client has abnormal amounts of circulating thyronine (T3) and thyroxine (T4). The nurse plans care for the client, anticipating that he or she may have a deficiency of which dietary elements? A. Iodine B. Calcium C. Phosphorus D. Magnesium

A. Iodine Adequate dietary iodine is needed to produce T3 and T4. The other requirements for adequate T3 and T4 production are an intact thyroid gland and a functional hypothalamus-pituitary-thyroid feedback system. Options 2, 3, and 4 are not directly related to the function of T3 and T4.

The nurse is caring for a postoperative parathyroidectomy client. Which client complaint would indicate that a life-threatening complication may be developing, requiring notification of the health care provider immediately? A. Laryngeal stridor B. Abdominal cramps C. Difficulty in voiding D. Mild to moderate incisional pain

A. Laryngeal stridor During the postoperative period, the nurse carefully observes the client for signs of hemorrhage, which causes swelling and compression of adjacent tissue. Laryngeal stridor is a harsh, high-pitched sound heard on inspiration and expiration; stridor is caused by compression of the trachea, leading to respiratory distress. Stridor is an acute emergency situation that requires immediate attention to avoid complete obstruction of the airway. Options 2, 3, and 4 do not identify signs of a life-threatening complication.

The nurse is developing a plan of care for a client with Addison's disease. The nurse has identified a problem of risk for deficient fluid volume and identifies nursing interventions that will prevent this occurrence. Which nursing interventions should the nurse include in the plan of care? Select all that apply. A. Monitor for changes in mentation. B. Encourage an intake of low-protein foods. C. Encourage an intake of low-sodium foods. D. Encourage fluid intake of at least 3000 mL per day. E. Monitor vital signs, skin turgor, and intake and output.

A. Monitor for changes in mentation. D. Encourage fluid intake of at least 3000 mL per day. E. Monitor vital signs, skin turgor, and intake and output. The client at risk for deficient fluid volume should be encouraged to eat regular meals and snacks and to increase intake of sodium, protein, and complex carbohydrates. Oral replacement of sodium losses is necessary, and maintenance of adequate blood glucose levels is required. Options 1, 4, and 5 are appropriate interventions for the client at risk for a deficient fluid volume.

A nurse is reviewing the health care provider's prescriptions for a client diagnosed with hypothyroidism. Which medication prescription should the nurse question and verify? A. Morphine sulfate B. Docusate sodium (Colace) C. Acetaminophen (Tylenol) D. Levothyroxine sodium (Synthroid)

A. Morphine sulfate Medications are administered very cautiously to the client with hypothyroidism because of altered metabolism and excretion and depressed metabolic rate and respiratory status. Morphine sulfate would further depress bodily functions. Hormone replacement with levothyroxine sodium (Synthroid), a thyroid hormone, is a component of therapy. Stool softeners, such as docusate sodium (Colace), are prescribed to prevent constipation. Acetaminophen (Tylenol) can be taken.

The nurse is monitoring a client newly diagnosed with diabetes mellitus for signs of complications. Which sign, if exhibited in the client, would indicate hyperglycemia? A. Polyuria B. Diaphoresis C. Hypertension D. Increased pulse rate

A. Polyuria Classic symptoms of hyperglycemia include polydipsia, polyuria, and polyphagia. Diaphoresis may occur in hypoglycemia. Options 2, 3, and 4 are not signs of hyperglycemia.

A nurse is reviewing the assessment findings for a client who was admitted to the hospital with a diagnosis of diabetes insipidus. The nurse understands that which manifestations are associated with this disorder? Select all that apply. A. Polyuria B. Polydipsia C. Concentrated urine D. Complaints of excessive thirst E. Specific gravity lower than 1.005

A. Polyuria B. Polydipsia D. Complaints of excessive thirst E. Specific gravity lower than 1.005 A triad of clinical symptoms including polyuria, polydipsia, and excessive thirst often occurs suddenly in the client with diabetes insipidus. The urine is dilute, with a specific gravity lower than 1.005, and the urine osmolality is low (50 to 200 mOsm/L).

The nurse should include which interventions in the plan of care for a client with hypothyroidism? Select all that apply. A. Provide a warm environment for the client. B. Instruct the client to consume a low-fat diet. C. A thyroid-releasing inhibitor will be prescribed. D. Encourage the client to consume a well-balanced diet. E. Instruct the client that thyroid replacement therapy will be needed. F. Instruct the client that episodes of chest pain are expected to occur.

A. Provide a warm environment for the client. B. Instruct the client to consume a low-fat diet. D. Encourage the client to consume a well-balanced diet. E. Instruct the client that thyroid replacement therapy will be needed. The clinical manifestations of hypothyroidism are the result of decreased metabolism caused by low levels of thyroid hormone. Interventions are aimed at replacement of the hormones and measures to support the signs and symptoms related to a decreased metabolism. The client often has cold intolerance and requires a warm environment. The nurse encourages the client to consume a well-balanced diet that is low in fat for weight reduction and high in fluids and high-fiber foods to prevent constipation. Iodine preparations are used to treat hyperthyroidism. Iodine preparations decrease blood flow through the thyroid gland and reduce the production and release of thyroid hormone. Thyroid replacement will be needed. The client would notify the health care provider if chest pain occurs because it could be an indication of overreplacement of thyroid hormone.

A client with diabetes mellitus is being discharged following treatment for hyperglycemic hyperosmolar state (HHS) precipitated by acute illness. The client tells the nurse, "will call the health care provider (HCP) the next time I can't eat for more than a day or so." Which statement reflects the most appropriate analysis of this client's level of knowledge? A. The client needs immediate education before discharge. B. The client requires follow-up teaching regarding the administration of oral antidiabetics. C. The client's statement is inaccurate, and he or she should be scheduled for outpatient diabetic counseling. D. The client's statement is inaccurate, and he or she should be scheduled for educational home health visits.

A. The client needs immediate education before discharge. If the client becomes ill and cannot retain fluids or food for a period of 4 hours, the HCP should be notified. The client's statement in this question indicates a need for immediate education to prevent HHNS, a life-threatening emergency. Although all other options may be true, the most appropriate analysis is that the client requires immediate education.

A hospitalized client is diagnosed with type 1 diabetes mellitus. The nurse plans care for the client, understanding that which factors are likely causes of the beta cell destruction that accompanies this disorder? Select all that apply. A. Viruses B. Genetic factors C. Autoimmune factors D. Human leukocyte antigen (HLA) E. Primary failure of glucagon secretion

A. Viruses B. Genetic factors C. Autoimmune factors D. Human leukocyte antigen (HLA) Viruses and autoimmune factors are thought to play a role in the development of type 1 diabetes mellitus. Other causes of type 1 diabetes mellitus include genetic factors, specifically the presence of the human leukocyte antigen (HLA). This factor is found in many clients with type 1 diabetes mellitus. The problem with type 1 diabetes mellitus is destruction of the beta cells. It is not caused by a primary failure of glucagon secretion.

The nurse is caring for a client with pheochromocytoma who is scheduled for adrenalectomy. In the preoperative period, what should the nurse monitor as the priority? A. Vital signs B. Intake and output C. Blood urea nitrogen results D. Urine for glucose and ketones

A. Vital signs Pheochromocytoma is a catecholamine-producing tumor. Hypertension is the hallmark of pheochromocytoma. Severe hypertension can precipitate a stroke or sudden blindness. Although all the options are accurate nursing interventions for the client with pheochromocytoma, the priority nursing action is to monitor the vital signs, particularly the blood pressure.

The home care nurse is visiting a client newly diagnosed with diabetes mellitus. The client tells the nurse that he is planning to eat a dinner meal at a local restaurant this week. He asks the nurse if eating at a restaurant will affect diabetic control and if this is allowed. Which nursing response is most appropriate? A. "You are not allowed to eat in restaurants." B. "You should order a half-portion meal and have fresh fruit for dessert." C. "If you plan to eat in a restaurant, you need to skip the lunchtime meal." D. "You should increase your daily dose of insulin by half on the day that you plan to eat in the restaurant."

B. "You should order a half-portion meal and have fresh fruit for dessert." Clients with diabetes mellitus are instructed to make adjustments in their total daily intake to plan for meals at restaurants or parties. Some useful strategies include ordering a half-portion, salads with dressing on the side, fresh fruit for dessert, and baked or steamed entrées. Clients are not instructed to skip meals or increase their prescribed insulin dosage.

The nursing instructor asks a nursing student to identify the risk factors associated with the development of thyrotoxicosis. The student demonstrates understanding of the risk factors by identifying an increased risk for thyrotoxicosis in which client? A. A client with hypothyroidism B. A client with Graves' disease who is having surgery C. A client with diabetes mellitus scheduled for a diagnostic test D. A client with diabetes mellitus scheduled for débridement of a foot ulcer

B. A client with Graves' disease who is having surgery Thyrotoxicosis usually is seen in clients with Graves' disease in whom the symptoms are precipitated by a major stressor. This complication typically occurs during periods of severe physiological or psychological stress such as trauma, sepsis, delivery, or major surgery. It also must be recognized as a potential complication after thyroidectomy. The client conditions in options 1, 3, and 4 are not associated with thyrotoxicosis.

The nurse is performing an assessment on a client with a diagnosis of hyperthyroidism. Which assessment finding should the nurse expect to note in this client? A. Dry skin B. Bulging eyeballs C. Periorbital edema D. Coarse facial features

B. Bulging eyeballs Hyperthyroidism is clinically manifested by goiter (increase in the size of the thyroid gland) and exophthalmos (bulging eyeballs). Other clinical manifestations include nervousness, fatigue, weight loss, muscle cramps, and heat intolerance. Additional signs found in this disorder include tachycardia; shortness of breath; excessive sweating; fine muscle tremors; thin, silky hair and thin skin; infrequent blinking; and a staring appearance.

A client with diabetes mellitus demonstrates acute anxiety when first admitted to the hospital for the treatment of hyperglycemia. What is the most appropriate intervention to decrease the client's anxiety? A. Administer a sedative. B. Convey empathy, trust, and respect toward the client. C. Ignore the signs and symptoms of anxiety so that they will soon disappear. D. Make sure that the client knows all the correct medical terms to understand what is happening.

B. Convey empathy, trust, and respect toward the client. The appropriate intervention is to address the client's feelings related to the anxiety. Administering a sedative is not the most appropriate intervention. The nurse should not ignore the client's anxious feelings. A client will not relate to medical terms, particularly when anxiety exists.

A client is diagnosed with Cushing's syndrome. The nurse plans care, knowing that this client has an excess of which substances? A. Calcium B. Cortisol C. Epinephrine D. Norepinephrine

B. Cortisol Cushing's syndrome is characterized by an excess of cortisol, a glucocorticoid. Glucocorticoids are produced by the adrenal cortex. Calcium is unrelated to this disorder. Epinephrine and norepinephrine are produced by the adrenal medulla.

The nurse is preparing a plan of care for a client with diabetes mellitus who has hyperglycemia. The nurse places highest priority on which client problem? A. Lack of knowledge B. Inadequate fluid volume C. Compromised family coping D. Inadequate consumption of nutrients

B. Inadequate fluid volume An increased blood glucose level will cause the kidneys to excrete the glucose in the urine. This glucose is accompanied by fluids and electrolytes, causing an osmotic diuresis leading to dehydration. This fluid loss must be replaced when it becomes severe. Options 1, 3, and 4 are not related specifically to the subject of the question.

The nurse is providing instructions to a client newly diagnosed with diabetes mellitus. The nurse gives the client a list of the signs of hyperglycemia. Which specific sign of this complication should be included on the list? A. Shakiness B. Increased thirst C. Profuse sweating D. Decreased urine output

B. Increased thirst The classic signs of hyperglycemia include polydipsia, polyuria, and polyphagia. Profuse sweating and shakiness would be noted in a hypoglycemic condition.

A client with diabetes mellitus is at risk for a serious metabolic disorder from the breakdown of fats for conversion to glucose. The nurse plans care for the client, knowing that pathological fat metabolism is occurring if the client has elevated levels of which substance? A. Glucose B. Ketones C. Glucagon D. Lactate dehydrogenase

B. Ketones Ketones are a by-product of fat metabolism. When this process occurs to an extreme, the resulting condition is called ketoacidosis. Options 1, 3, and 4 are not associated with the breakdown of fats.

The nurse teaches a client with diabetes mellitus about differentiating between hypoglycemia and ketoacidosis. The client demonstrates an understanding of the teaching by stating that a form of glucose should be taken if which symptoms develop? Select all that apply. A. Polyuria B. Shakiness C. Palpitations D. Blurred vision E. Lightheadedness F. Fruity breath odor

B. Shakiness C. Palpitations E. Lightheadedness Shakiness, palpitations, and lightheadedness are signs of hypoglycemia and would indicate the need for food or glucose. Polyuria, blurred vision, and a fruity breath odor are signs of hyperglycemia.

The nurse has developed a postoperative plan of care for a client who had a thyroidectomy and documents that the client is at risk for developing an ineffective breathing pattern. Which nursing intervention should the nurse include in the plan of care? A. Maintain a supine position. B. Monitor neck circumference every 4 hours. C. Maintain a pressure dressing on the operative site. D. Encourage deep breathing exercises and vigorous coughing exercises

B. Monitor neck circumference every 4 hours. After thyroidectomy neck circumference is monitored every 4 hours to assess for the occurrence of postoperative edema. The client should be placed in an upright position to facilitate air exchange. A pressure dressing is not placed on the operative site because it may restrict breathing. The nurse should monitor the dressing closely and should loosen the dressing if necessary. The nurse should assist the client with deep breathing exercises, but coughing is minimized to prevent tissue damage and stress to the incision.

A client has overactivity of the thyroid gland. The nurse plans care, knowing that the client will experience which effects from this hormonal excess? A. Weight gain B. Nutritional deficiencies C. Low blood glucose levels D. Increased body fat stores

B. Nutritional deficiencies Although the client may experience an increased appetite with overactivity of the thyroid gland, food intake does not meet energy demands, and nutritional deficiencies can develop. Weight loss occurs as a result of the increased metabolic activity. Glucose tolerance is decreased, and the client experiences hyperglycemia. Overactivity of the thyroid gland also causes increased metabolism, including fat metabolism. This leads to decreased levels of fat in the bloodstream, including cholesterol, and decreased body fat stores.

The nurse is caring for a client who is 2 days postoperative following an abdominal hysterectomy. The client has a history of diabetes mellitus and has been receiving regular insulin according to capillary blood glucose testing four times a day. A carbohydrate-controlled diet has been prescribed but the client has been complaining of nausea and is not eating. On entering the client's room, the nurse finds the client to be confused and diaphoretic. Which action is most appropriate at this time? A. Call a code to obtain needed assistance immediately. B. Obtain a capillary blood glucose level and perform a focused assessment. C. Ask the unlicensed assistive personnel (UAP) to stay with the client while obtaining 15 to 30 g of a carbohydrate snack for the client to eat. D. Stay with the client and ask the UAP to call the health care provider (HCP) for a prescription for intravenous 50% dextrose.

B. Obtain a capillary blood glucose level and perform a focused assessment. Diaphoresis and confusion are signs of moderate hypoglycemia. A likely cause of the client's change in condition could be related to the administration of insulin without the client eating enough food. However, an assessment is necessary to confirm the presence of hypoglycemia. The nurse would obtain a capillary blood glucose level to confirm the hypoglycemia and perform a focused assessment to determine the extent and cause of the client's condition. Once hypoglycemia is confirmed, the nurse stays with the client and asks the unlicensed assistive personnel (UAP) to obtain the appropriate carbohydrate snack. A code is called if the client is not breathing or if the heart is not beating.

The nurse is completing an assessment on a client who is being admitted for a diagnostic workup for primary hyperparathyroidism. Which client complaint would be characteristic of this disorder? A. Diarrhea B. Polyuria C. Polyphagia D. Weight gain

B. Polyuria Hypercalcemia is the hallmark of hyperparathyroidism. Elevated serum calcium levels produce osmotic diuresis and thus polyuria. This diuresis leads to dehydration (weight loss rather than weight gain). Options 1, 3, and 4 are not associated with hyperparathyroidism. Some gastrointestinal symptoms include anorexia, nausea, vomiting,and, constipation.

The nurse caring for a client with a diagnosis of hypoparathyroidism reviews the laboratory results of blood tests for this client and notes that the calcium level is extremely low. The nurse should expect to note which on assessment of the client? A. Unresponsive pupils B. Positive Trousseau's sign C. Negative Chvostek's sign D. Hyperactive bowel sounds

B. Positive Trousseau's sign Hypoparathyroidism is related to a lack of parathyroid hormone secretion or a decreased effectiveness of parathyroid hormone on target tissues. The end result of this disorder is hypocalcemia. When serum calcium levels are critically low, the client may exhibit Chvostek's and Trousseau's signs, which indicate potential tetany. Options 1, 3, and 4 are not related to the presence of hypocalcemia.

The nurse is caring for a client admitted to the hospital with uncontrolled type 1 diabetes mellitus. In the event that diabetic ketoacidosis (DKA) does occur, the nurse anticipates that which medication would most likely be prescribed? A. Glucagon B. Regular insulin C. Glyburide (DiaBeta) D. Neutral protamine Hagedorn (NPH) insulin

B. Regular insulin Giving regular insulin by the intravenous route is the treatment of choice for DKA. A short-acting insulin is the only insulin that can be given intravenously because it can be titrated to the client's blood glucose levels. Glucagon is used to treat hypoglycemia because it increases blood glucose levels, and glyburide is an oral hypoglycemic agent used to treat type 2 diabetes mellitus; both agents are inappropriate. NPH insulin is an intermediate-acting insulin and therefore is not appropriate for treatment of DKA.

The nurse is caring for a client after hypophysectomy and notes clear nasal drainage from the client's nostril. The nurse should take which initial action? A. Lower the head of the bed. B. Test the drainage for glucose. C. Obtain a culture of the drainage. D. Continue to observe the drainage.

B. Test the drainage for glucose. After hypophysectomy, the client should be monitored for rhinorrhea, which could indicate a cerebrospinal fluid leak. If this occurs, the drainage should be collected and tested for the presence of cerebrospinal fluid. The head of the bed should not be lowered to prevent increased intracranial pressure. Clear nasal drainage would not indicate the need for a culture. Continuing to observe the drainage without taking action could result in a serious complication.

A client with type 1 diabetes mellitus calls the nurse to report recurrent episodes of hypoglycemia with exercising. Which statement by the client indicates an inadequate understanding of the peak action of NPH insulin and exercise? A. "The best time for me to exercise is after I eat." B. "The best time for me to exercise is after breakfast." C. "The best time for me to exercise is mid- to late afternoon." D. "The best time for me to exercise is after my morning snack."

C. "The best time for me to exercise is mid- to late afternoon." A hypoglycemic reaction may occur in response to increased exercise. Clients should avoid exercise during the peak time of insulin. NPH insulin peaks at 4 to 12 hours; therefore, afternoon exercise takes place during the peak of the medication. Options 1, 2, and 4 do not address peak action times.

The nurse is reviewing the laboratory test results for a client with a diagnosis of Cushing's syndrome. Which laboratory finding would the nurse expect to note in this client? A. A platelet count of 200,000 cells/mm3 B. A blood glucose level of 110 mg/dL C. A potassium (K+) level of 3.1 mEq/L D. A white blood cell (WBC) count of 6000 cells/mm3

C. A potassium (K+) level of 3.1 mEq/L The client with Cushing's syndrome experiences hypokalemia, hyperglycemia, an elevated WBC count, and elevated plasma cortisol and adrenocorticotropic hormone levels. These abnormalities are caused by the effects of excess glucocorticoids and mineralocorticoids in the body. The laboratory values listed in options 1, 2, and 4 would not be noted in the client with Cushing's syndrome.

The nurse is reviewing the postoperative prescriptions for a client who had a transsphenoidal hypophysectomy. Which health care provider's (HCP) prescriptions, if noted on the record, would indicate the need for clarification? A. Assess vital signs and neurological status. B. Instruct the client to avoid blowing his nose. C. Apply a loose dressing if any clear drainage is noted. D. Instruct the client about the need for a Medic-Alert bracelet.

C. Apply a loose dressing if any clear drainage is noted. The nurse should observe for clear nasal drainage; constant swallowing; and a severe, persistent, generalized, or frontal headache. These signs and symptoms indicate cerebrospinal fluid leak into the sinuses. If clear drainage is noted after this procedure, the HCP needs to be notified. Options 1, 2, and 4 indicate appropriate postoperative interventions.

The nurse is providing instructions regarding home care measures to a client with diabetes mellitus and instructs the client about the causes of hypoglycemia. The nurse determines that additional instruction is needed if the client identifies which as a cause of hypoglycemia? A. Omitted meals B. Increased intensity of activity C. Decreased daily insulin dosage D. Inadequate amount of fluid intake

C. Decreased daily insulin dosage Decreasing the dose of insulin will lead to hyperglycemia. Causes for hypoglycemic reactions include delayed consumption of meals and lack of necessary amounts of food. Other causes include the administration of excessive insulin or oral hypoglycemic medications, vomiting associated with illness, and strenuous exercise, which may potentiate the action of insulin.

A client with a diagnosis of diabetic ketoacidosis (DKA) is being treated in the emergency department. Which findings would the nurse expect to note as confirming this diagnosis? Select all that apply. A. Increase in pH B. Comatose state C. Deep, rapid breathing D. Decreased urine output E. Elevated blood glucose level F. Low plasma bicarbonate level

C. Deep, rapid breathing E. Elevated blood glucose level F. Low plasma bicarbonate level In DKA, the arterial pH is lower than 7.35, plasma bicarbonate is lower than 15 mEq/L, the blood glucose level is higher than 250 mg/dL, and ketones are present in the blood and urine. The client would be experiencing polyuria, and Kussmaul's respirations (deep and rapid breathing pattern) would be present. A comatose state may occur if DKA is not treated, but coma would not confirm the diagnosis.

The nurse is monitoring a client with diabetes mellitus for signs of hypoglycemia. Which manifestations are associated with this complication? A. Slow pulse; lethargy; warm, dry skin B. Elevated pulse; lethargy; warm, dry skin C. Elevated pulse; shakiness; cool, clammy skin D. Slow pulse, confusion, increased urine output

C. Elevated pulse; shakiness; cool, clammy skin Signs and symptoms of mild hypoglycemia include tachycardia, shakiness, and cool, clammy skin. Options 1, 2, and 4 do not specify the manifestations of hypoglycemia.

The nurse is preparing a client with a new diagnosis of hypothyroidism for discharge. The nurse determines that the client understands discharge instructions if the client states that which symptoms are associated with this diagnosis? Select all that apply. A. Tremors B. Weight loss C. Feeling cold D. Loss of body hair E. Persistent lethargy F. Puffiness of the face

C. Feeling cold D. Loss of body hair E. Persistent lethargy F. Puffiness of the face Feeling cold, hair loss, lethargy, and facial puffiness are signs of hypothyroidism. Tremors and weight loss are signs of hyperthyroidism.

A nurse is assigned to the care of a client who has an altered production of cortisol. The nurse anticipates that the client is experiencing difficulty with synthesis of which type of substance? A. Androgens B. Catecholamines C. Glucocorticoids D. Mineralocorticoids

C. Glucocorticoids Cortisol is a glucocorticoid, which is produced by the adrenal cortex. Androgens and mineralocorticoids are other substances produced by the adrenal cortex. Catecholamines (epinephrine and norepinephrine) are produced by the adrenal medulla.

A client has been diagnosed with pheochromocytoma. The nurse plans care, knowing that the client will exhibit which effect based on the pathophysiology of this disorder? A. Water loss B. Bradycardia C. Hypertension D. Decreased cardiac output

C. Hypertension The client with pheochromocytoma has a benign or malignant tumor in the adrenal medulla. Because the medulla secretes epinephrine and norepinephrine, the client will exhibit signs related to excesses of these catecholamines, including tachycardia, increased cardiac output, and increased blood pressure. Vasoconstriction of the renal arteries triggers the renin-angiotensin system, resulting in water reabsorption and retention.

The emergency department nurse is preparing a plan for initial care of a client with a diagnosis of hyperglycemic hyperosmolar state (HHS). The nurse understands that the hyperglycemia associated with this disorder results from which occurrence? A. Increased use of glucose B. Overproduction of insulin C. Increased production of glucose D. Increased osmotic movement of water

C. Increased production of glucose Hyperglycemia results from decreased use and increased production of glucose. Increased use of glucose and overproduction of insulin would most likely cause hypoglycemia. Option 4 is incorrect.

The nurse is providing instructions regarding insulin administration for a client newly diagnosed with diabetes mellitus. The health care provider has prescribed a mixture of Humulin N and Humulin R insulin. The nurse should instruct the client that which is the first step in this procedure? A. Draw up the correct dosage of Humulin N insulin into the syringe. B. Draw up the correct dosage of Humulin R insulin into the syringe. C. Inject air equal to the amount of Humulin N prescribed into the vial of Humulin N insulin. D. Inject air equal to the amount of Humulin R prescribed into the vial of Humulin R insulin.

C. Inject air equal to the amount of Humulin N prescribed into the vial of Humulin N insulin. The initial step in preparing an injection of insulin that is a mixture of Humulin N and regular is to inject air into the Humulin N bottle equal to the amount of insulin prescribed. The client would then be instructed to next inject an amount of air equal to the amount of prescribed insulin into the Humulin R bottle. The regular insulin would then be withdrawn, followed by the Humulin N insulin. Contamination of regular insulin with Humulin N insulin will convert part of the Humulin R insulin into a longer-acting form.

The nurse should include which interventions in the plan of care for a client with hypothyroidism? Select all that apply. A. Provide a cool environment for the client. B. Instruct the client to consume a high-fat diet. C. Instruct the client about thyroid replacement therapy. D. Encourage the client to consume fluids and high-fiber foods in the diet. E. Inform the client that iodine preparations will be prescribed to treat the disorder. F. Instruct the client to contact the health care provider (HCP) if episodes of chest pain occur.

C. Instruct the client about thyroid replacement therapy. D. Encourage the client to consume fluids and high-fiber foods in the diet. F. Instruct the client to contact the health care provider (HCP) if episodes of chest pain occur. The clinical manifestations of hypothyroidism are the result of decreased metabolism from low levels of thyroid hormone. Interventions are aimed at replacement of the hormone and providing measures to support the signs and symptoms related to decreased metabolism. The client often has cold intolerance and requires a warm environment. The nurse encourages the client to consume a well-balanced diet that is low in fat for weight reduction and high in fluids and high-fiber foods to prevent constipation. Iodine preparations may be used to treat hyperthyroidism. Iodine preparations decrease blood flow through the thyroid gland and reduce the production and release of thyroid hormone; they are not used to treat hypothyroidism. The client is instructed to notify the HCP if chest pain occurs because it could be an indication of overreplacement of thyroid hormone.

A client is admitted to a hospital with a diagnosis of diabetic ketoacidosis (DKA). The initial blood glucose level was 950 mg/dL. A continuous intravenous infusion of short-acting insulin is initiated, along with intravenous rehydration with normal saline. The serum glucose level is now 240 mg/dL. The nurse would next prepare to administer which item? A. Ampule of 50% dextrose B. NPH insulin subcutaneously C. Intravenous fluids containing dextrose D. Phenytoin (Dilantin) for the prevention of seizures

C. Intravenous fluids containing dextrose During management of DKA, when the blood glucose level falls to 250 to 300 mg/dL, the infusion rate is reduced and a dextrose solution is added to maintain a blood glucose level of about 250 mg/dL, or until the client recovers from ketosis. Fifty percent dextrose is used to treat hypoglycemia. NPH insulin is not used to treat DKA. Phenytoin (Dilantin) is not a usual treatment measure for DKA.

A client is brought to the emergency department in an unresponsive state, and a diagnosis of hyperglycemic hyperosmolar state (HHS) is made. The nurse would immediately prepare to initiate which anticipated health care provider's prescription? A. Endotracheal intubation B. 100 units of NPH insulin C. Intravenous infusion of normal saline D. Intravenous infusion of sodium bicarbonate

C. Intravenous infusion of normal saline The primary goal of treatment in hyperglycemic hyperosmolar state (HHS) is to rehydrate the client to restore fluid volume and to correct electrolyte deficiency. Intravenous fluid replacement is similar to that administered in diabetic ketoacidosis (DKA) and begins with IV infusion of normal saline. Regular insulin, not NPH insulin, would be administered. The use of sodium bicarbonate to correct acidosis is avoided because it can precipitate a further drop in serum potassium levels. Intubation and mechanical ventilation are not required to treat HHS.

The nurse is caring for a client after thyroidectomy. The client expresses concern about the postoperative voice hoarseness she is experiencing and asks if the hoarseness will subside. The nurse should provide the client with which information? A. The hoarseness is permanent. B. It indicates nerve damage. C. It is normal during this time and will subside. D. It will worsen before it subsides, which may take 6 months.

C. It is normal during this time and will subside. Hoarseness in the postoperative period usually is the result of laryngeal pressure or edema and will resolve within a few days. The client should be reassured that the effects are transitory. Options 1, 2, and 4 are incorrect.

After several diagnostic tests, a client is diagnosed with diabetes insipidus. The nurse performs an assessment on the client, knowing that which symptom is most indicative of this disorder? A. Fatigue B. Diarrhea C. Polydipsia D. Weight gain

C. Polydipsia Diabetes insipidus is characterized by hyposecretion of antidiuretic hormone, and the kidney tubules fail to reabsorb water. Polydipsia and polyuria are classic symptoms of diabetes insipidus. The urine is pale, and the specific gravity is low. Anorexia and weight loss occur. Option 1 is a vague symptom. Options 2 and 4 are not specific to this disorder.

A client has just been admitted to the nursing unit following thyroidectomy. Which assessment is the priority for this client? A. Hypoglycemia B. Level of hoarseness C. Respiratory distress D. Edema at the surgical site

C. Respiratory distress Thyroidectomy is the removal of the thyroid gland, which is located in the anterior neck. It is very important to monitor airway status as any swelling to the surgical site could cause respiratory distress. Although all the options are important for the nurse to monitor, the priority nursing action is to monitor the airway.

A client with type 1 diabetes mellitus is to begin an exercise program, and the nurse is providing instructions regarding the program. Which instruction should the nurse include in the teaching plan? A. Try to exercise before mealtimes. B. Administer insulin after exercising. C. Take a blood glucose test before exercising. D. Exercise is best performed during peak times of insulin.

C. Take a blood glucose test before exercising. A blood glucose test performed before exercising provides the client with information regarding the need to consume a snack before exercising. Exercising during the peak times of insulin or before mealtime places the client at risk for hypoglycemia. Insulin should be administered as prescribed.

The nurse performs a physical assessment on a client with type 2 diabetes mellitus. Findings include a fasting blood glucose level of 120 mg/dL, temperature of 101° F, pulse of 88 beats/minute, respirations of 22 breaths/minute, and blood pressure of 100/72 mm Hg. Which assessment would be of most concern to the nurse? A. Pulse B. Respiration C. Temperature D. Blood pressure

C. Temperature An elevated temperature may indicate infection. Infection is a leading cause of hyperglycemic hyperosmolar state or diabetic ketoacidosis. The other findings noted in the question are within normal limits.

The nurse is monitoring a client for signs of hypocalcemia after thyroidectomy. Which sign/symptom, if noted in the client, would most likely indicate the presence of hypocalcemia? A. Bradycardia B. Flaccid paralysis C. Tingling around the mouth D. Absence of Chvostek's sign

C. Tingling around the mouth After thyroidectomy the nurse assesses the client for signs of hypocalcemia and tetany. Early signs include tingling around the mouth and in the fingertips, muscle twitching or spasms, palpitations or arrhythmias, and Chvostek's and Trousseau's signs. Options 1, 2, and 4 are not signs of hypocalcemia.

The nursing instructor asks a student to describe the pathophysiology that occurs in Cushing's disease. Which statement by the student indicates an accurate understanding of this disorder? A. "Cushing's disease results from an oversecretion of insulin." B. "Cushing's disease results from an undersecretion of corticotropic hormones." C. "Cushing's disease results from an undersecretion of mineralocorticoid hormones." D. "Cushing's disease results from an increased pituitary secretion of adrenocorticotropic hormone."

D. "Cushing's disease results from an increased pituitary secretion of adrenocorticotropic hormone." Cushing's disease is a metabolic disorder characterized by abnormally increased secretion (endogenous) of cortisol, caused by increased amounts of adrenocorticotropic hormone (ACTH) secreted by the pituitary gland. Addison's disease is characterized by the hyposecretion of adrenal cortex hormones (glucocorticoids and mineralocorticoids) from the adrenal gland, resulting in deficiency of the corticosteroid hormones. Options 1, 2, and 3 are inaccurate regarding Cushing's disease.

The home care nurse visits a client with a diagnosis of hyperparathyroidism who is taking furosemide (Lasix) and provides dietary instructions to the client. Which statement by the client indicates a need for additional instruction? A. "I need to eat foods high in potassium." B. "I need to drink at least 2 to 3 L of fluid daily." C. "I need to eat small, frequent meals and snacks if nauseated." D. "I need to increase my intake of dietary items that are high in calcium."

D. "I need to increase my intake of dietary items that are high in calcium." The aim of treatment in the client with hyperparathyroidism is to increase the renal excretion of calcium and decrease gastrointestinal absorption and bone resorption of calcium. Dietary restriction of calcium may be used as a component of therapy. The client should eat foods high in potassium, especially if the client is taking furosemide. Options 2 and 3 also are appropriate instructions for the client.

The nurse has provided instructions for measuring blood glucose levels to a client newly diagnosed with diabetes mellitus who will be taking insulin. The client demonstrates understanding of the instructions by identifying which method as the best method for monitoring blood glucose levels? A. "I will check my blood glucose level every day at 5:00 pm." B. "I will check my blood glucose level 1 hour after each meal." C. "I will check my blood glucose level 2 hours after each meal." D. "I will check my blood glucose level before each meal and at bedtime."

D. "I will check my blood glucose level before each meal and at bedtime." The most effective and accurate measure for testing blood glucose is to test the level before each meal and at bedtime. If possible and feasible, testing should be done during the nighttime hours. Checking the level after the meal will provide an inaccurate assessment of diabetes control. Checking the level once daily will not provide enough data related to control the diabetes mellitus.

A client with type 1 diabetes mellitus is having trouble remembering the types, duration, and onset of the action of insulin. The client tells the nurse that the family members have not been supportive. Which response by the nurse is best? A. "What is it that you don't understand?" B. "You can't always depend on your family to help." C. "It's not really necessary for you to remember this." D. "Let me go over the types of insulin with you again."

D. "Let me go over the types of insulin with you again." Reinforcement of knowledge and behaviors is vital to the success of the client's self-care. All other options do not address the need for client instructions and are not therapeutic responses.

The nurse is interviewing a client with type 2 diabetes mellitus. Which statement by the client indicates an understanding of the treatment for this disorder? A. "I take oral insulin instead of shots." B. "By taking these medications, I am able to eat more." C. "When I become ill, I need to increase the number of pills I take." D. "The medications I'm taking help release the insulin I already make."

D. "The medications I'm taking help release the insulin I already make." Clients with type 2 diabetes mellitus have decreased or impaired insulin secretion. Oral hypoglycemic agents are given to these clients to facilitate glucose uptake. Insulin injections may be given during times of stress-induced hyperglycemia. Oral insulin is not available because of the breakdown of the insulin by digestion. Options 1, 2, and 3 are incorrect.

The nurse is performing an assessment on a client with pheochromocytoma. Which assessment data would indicate a potential complication associated with this disorder? A. A coagulation time of 5 minutes B. A urinary output of 50 mL/hour C. A blood urea nitrogen level of 20 mg/dL D. A heart rate that is 90 beats/minute and irregular

D. A heart rate that is 90 beats/minute and irregular The complications associated with pheochromocytoma include hypertensive retinopathy and nephropathy, myocarditis, increased platelet aggregation, and stroke. Death can occur from shock, stroke, kidney failure, dysrhythmias, or dissecting aortic aneurysm. An irregular heart rate indicates the presence of a dysrhythmia. A coagulation time of 5 minutes is normal. A urinary output of 50 mL/hour is an adequate output. A blood urea nitrogen level of 20 mg/dL is a normal finding.

The nurse is performing an assessment on a client with a diagnosis of Cushing's syndrome. Which should the nurse expect to note on assessment of the client? A. Skin atrophy B. The presence of sunken eyes C. Drooping on one side of the face D. A rounded "moon-like" appearance to the face

D. A rounded "moon-like" appearance to the face With excessive secretion of adrenocorticotropic hormone (ACTH) and chronic corticosteroid use, the person with Cushing's syndrome develops a rounded moon-like face; prominent jowls; red cheeks; and hirsutism on the upper lip, lower cheek, and chin. Options 1, 2, and 3 are not associated with the assessment findings in Cushing's syndrome.

The nurse is reviewing the health care provider (HCP) prescriptions for a client with a diagnosis of diabetes mellitus who has been hospitalized for treatment of an infected foot ulcer. The nurse expects to note which finding in the HCP prescriptions? A. A decreased-calorie diet B. An increased-calorie diet C. A decreased amount of NPH daily insulin D. An increased amount of NPH daily insulin

D. An increased amount of NPH daily insulin Infection is a physiological stressor that can cause an increase in the level of epinephrine in the body. An increase in epinephrine causes an increase in blood glucose levels. When the client is under stress, such as when an infection is present, an increase in the dose of insulin will be required to facilitate the transport of excess glucose into the cells. The client will not necessarily need an adjustment in the daily diet.

A client has a tumor that is interfering with the function of the hypothalamus. The nurse expects that which clinical problem will be exhibited by the client? A. Melatonin excess or deficit B. Glucocorticoid excess or deficit C. Mineralocorticoid excess or deficit D. Antidiuretic hormone (ADH) excess or deficit

D. Antidiuretic hormone (ADH) excess or deficit The hypothalamus exerts an influence on both the anterior and the posterior pituitary gland. Abnormalities can result in excess or deficit of substances normally mediated by the pituitary. ADH could be affected by disease of the hypothalamus because the hypothalamus produces ADH and stores it in the posterior pituitary gland. The pineal gland is responsible for melatonin production. The adrenal cortex is responsible for the production of glucocorticoids and mineralocorticoids.

The nurse is developing a plan of care for a client who is scheduled for a thyroidectomy. The nurse focuses on psychosocial needs, knowing that which is likely to occur in the client? A. Infertility B. Gynecomastia C. Sexual dysfunction D. Body image changes

D. Body image changes Because of the location of the incision in the neck area, many clients are afraid of thyroid surgery for fear of having a visible large scar postoperatively. Having all or part of the thyroid gland removed will not cause the client to experience gynecomastia or hirsutism. Sexual dysfunction and infertility could occur if the entire thyroid is removed and the client is not placed on thyroid replacement medications.

The nurse has documented the problem of body image distortion for a client with a diagnosis of Cushing's syndrome. The nurse identifies nursing interventions related to this problem and includes these interventions in the plan of care. Which nursing intervention is inappropriate? A. Encourage client's expression of feelings. B. Assess the client's understanding of the disease process. C. Encourage family members to share their feelings about the disease process. D. Encourage the client to recognize that the body changes need to be dealt with.

D. Encourage the client to recognize that the body changes need to be dealt with. Encouraging the client to understand that the body changes that occur in this disorder need to be dealt with is an inappropriate nursing intervention. This option does not address the client's feelings. Options 1, 2, and 3 are appropriate because they address the client and family feelings regarding the disorder.

An external insulin pump is prescribed for a client with diabetes mellitus and the client asks the nurse about the functioning of the pump. The nurse bases the response on which information about the pump? A. Is timed to release programmed doses of short-duration or NPH insulin into the bloodstream at specific intervals B. Continuously infuses small amounts of NPH insulin into the bloodstream while regularly monitoring blood glucose levels C. Is surgically attached to the pancreas and infuses regular insulin into the pancreas, which in turn releases the insulin into the bloodstream D. Gives a small continuous dose of short-duration insulin subcutaneously, and the client can self-administer a bolus with an additional dose from the pump before each meal

D. Gives a small continuous dose of short-duration insulin subcutaneously, and the client can self-administer a bolus with an additional dose from the pump before each meal An insulin pump provides a small continuous dose of short-duration (rapid or short-acting) insulin subcutaneously throughout the day and night, and the client can self-administer a bolus with an additional dose from the pump before each meal as needed. Short-duration insulin is used in an insulin pump. An external pump is not attached surgically to the pancreas.

A client arrives in the hospital emergency department complaining of severe thirst and polyuria. The client tells the nurse that she has a history of diabetes mellitus. A blood glucose level is drawn, and the result is 685 mg/dL. Which intervention should the nurse anticipate to be initially prescribed for the client? A. Glucagon via the subcutaneous route B. Glyburide (DiaBeta) via the oral route C. Humulin N insulin via the subcutaneous route D. Humulin R insulin via the intravenous (IV) route

D. Humulin R insulin via the intravenous (IV) route The client is most likely in diabetic ketoacidosis (DKA). Humulin R insulin via the IV route is the preferred treatment for DKA. Humulin R insulin is a short-acting insulin and can be given intravenously; it is titrated to the client's high blood glucose levels. Glucagon is used to treat hypoglycemia, and glyburide is an oral hypoglycemia agent used to treat diabetes mellitus type 2. Humulin N insulin is an intermediate-acting insulin and is not appropriate for the emergency treatment of DKA.

A client with type 2 diabetes mellitus has a blood glucose level greater than 600 mg/dL and is complaining of polydipsia, polyuria, weight loss, and weakness. The nurse reviews the health care provider's documentation and expects to note which diagnosis? A. Hypoglycemia B. Pheochromocytoma C. Diabetic ketoacidosis (DKA) D. Hyperglycemic hyperosmolar nonketotic syndrome (HHNS)

D. Hyperglycemic hyperosmolar nonketotic syndrome (HHNS) HHNS is seen primarily in clients with type 2 diabetes mellitus, who experience a relative deficiency of insulin. The onset of signs and symptoms may be gradual. Manifestations may include polyuria, polydipsia, dehydration, mental status alterations, weight loss, and weakness. The clinical manifestations noted in the question are not signs of hypoglycemia. Pheochromocytoma is not related to these clinical manifestations. DKA typically occurs in type 1 diabetes mellitus.


Conjuntos de estudio relacionados

Chapter 57: Management of Patients with Burn Injury - ML3

View Set

Ch.45 Mgmnt of pts w/ oral esophageal disorders

View Set

Chapter 18 - Caring for Clients with Cancer, Chapter 18: Caring for Clients with Cancer Prep -U

View Set

Cervical Spine Anatomy and Pathology

View Set

Abeka grade 8 science reading quiz H

View Set